You are on page 1of 35

v. Performance of assisted vaginal deliveries.

Introduction vi. Removal of retained placental products.


vii. Manual removal of retained placenta.
viii.BeMONC facility shall consist of the core district
Maternal and Child Health hospital.
ix. For geographically isolated/disadvantaged
 refer to mother and child relationship to one another.
areas/ densely populated areas, the
 consideration of the entire family as well as the
designated BeMONC facilities are the following:
culture and socioeconomic environment as
Rural Health Unit, Barangay Health Station,
framework of the patient.
Lying-in Clinics and Birthing Homes.
 care of the woman and family throughout
x. Accessibility within 1 hour from residence or
pregnancy and childbirth and the health promotion
referring facility within the ILHZ (Inter-local
and illness care for the children and families.
Health Zones).
xi. Shall operate within 24 hours with 6 signal
Goal of Maternal and Child Health:
obstetric function.
i. To ensure that every expectant and nursing mother
xii. Shall have access to communication and
maintains good health, learns the art of child care,
transportation facilities to mobilize referrals.
has normal delivery and bears healthy child.
xiii.Staff composition: (1) Medical Doctor, (1)
ii. That every child grows up in a family unit with love
Registered Nurse, (1) Registered Midwife.
and security, in healthy surroundings, receives
 It also includes neonatal interventions which include
adequate nourishment, health supervision and
at the minimum:
efficient medical attention, and is taught the
i. Newborn resuscitation
elements of healthy living (Reyala, 2000).
ii. Provision of warmth
iii. Promotion and maintenance of optimum health of
iii. Referral
the women and newborn.
iv. Blood transfusion
iv. Promotion and maintenance of optimal family
health.
Comprehensive Emergency Obstetrics and Newborn
Care Facility (CeMONC)
Objectives of Maternal and Child Health:
 Refers to lifesaving services for emergency
i. Reduce maternal mortality and morbidity.
maternal and newborn conditions/complications as
ii. Reduce prenatal and neonatal mortality and
in Basic Emergency Obstetric and Newborn Care
morbidity.
plus the provision of surgical delivery and blood
iii. Regulate fertility so as to have wanted and healthy
bank services and other specialized obstetric
children when desired.
interventions.
iv. Provide basic maternal and child health care to all
mother and children.
Essential Health Services Available in the Health
v. Promote and protect health of mothers.
Care Facilities
vi. Promote and protect physical growth and psycho-
1. Antenatal Registration/Prenatal Care
social development of children.
 to reach all pregnant women, to give sufficient
Philosophy of Maternal and Child Health:
care.
i. Is community-centered.
 to ensure a healthy pregnancy and the birth of a
ii. Is research-centered.
full term healthy baby.
iii. Is based on nursing theory.
i. Normal Patients - following the initial evaluation
iv. Protects the rights of all family members.
they will be given healthy instructions and
v. Uses high degree of independent functioning.
counseling. This will include advice for prompt
vi. Places importance on promotion of health.
prenatal care examination.
ii. Patients with Mild Complications - a thorough
Maternal Neonatal and Child Health and Nutrition
iii. evaluation of the needs of patients with mild
Strategy (MNCHN)
complications will determine the frequency of
 It applies specific policies and actions for local
follow-up of these cases by the rural health
health systems to systematically address health
unit, city health clinic or puericulture center.
risks that lead to maternal and especially neonatal
iv. Patients with Potentially Serious Complications -
deaths which comprise half of the reported infant
these patients shall be referred to the most
mortality.
skilled
v. source of medical and hospital care. As a first
Basic Emergency Obstetrics and Newborn Care
choice they will be referred if at all possible for
(BeMONC)
continuing care or consultation. Second choice
 It refers to lifesaving services for emergency
will be followed carefully by the rural health
maternal and newborn conditions/complications
unit, city health clinic or puericulture center.
being provided by a health facility or professional to
include the following services:
First Prenatal Visit
i. Administration of parenteral oxytocic drugs.
 as early in pregnancy as possible, during the first
ii. Administration of dose of parenteral
trimester.
anticonvulsant.
Second Prenatal Visit
iii. Administration of parenteral antibiotics.
 during the second trimester.
iv. Administration of maternal steroids for preterm
Third and Subsequent Visits
labor.
 during the third trimester.  No previous cesarean section
 Imminent deliveries (those who are about to deliver
[And the three booster dose shots to complete the five and can no longer reach the nearest facility in time
doses following the recommended schedule provides full for delivery)
protection. The mother is then called as a “Fully  No premature rupture of membranes
Immunized Mother” (FIM).]  Adequate pelvis
 Abdominal enlargement is appropriate for age of
Micro-nutrient Supplementation gestation.
 It is necessary to prevent anemia, vitamin A
deficiency and other nutritional disorders. Home Delivery Kit
Vitamin A i. two pairs of clamps
 Dose: 10,000 IU ii. a pair of scissors
 Given a week starting on the 4th month of iii. antiseptic (may use 70% Povidone/Iodine)
pregnancy. iv. soap and hand brush
 Do not give it before the 4th month of pregnancy v. clean towel/piece of cloth
because it might cause congenital problems in the vi. flashlight
baby. vii. Sphygmomanometer
Iron viii. Stethoscope
 Dose: 60mg/400 ug tablet  Clean hands, clean surface and clean cord must be
 Schedule: Daily strictly followed to prevent infection.

Clean and Safe Delivery Guide for Home Delivery:


1. Check for emergency signs For registered patient
i. Unconsciousness  time when regular pains started, whether bag of
ii. Vaginal bleeding water ruptured or not, presence of absence of
iii. Severe abdominal bleeding vaginal discharges, bleeding, etc., whether mother
iv. Looks very ill moved her bowels and has urinated, fetal
v. Severe headache with visual disturbance movement felt by the mother or not, unusual
vi. Severe breathing difficulty symptoms such as bleeding, headache, spots
vii. Fever before eyes.
viii.Severe vomiting For unregistered patients
2. Made woman comfortable  get same information as for those registered
3. Assess the woman in labor patients and get medical and obstetric history.
i. LMP
ii. Number of pregnancy Delivery in Hospitals:
iii. Start of labor pains  Risk pregnancies should be advised to deliver in
iv. Age/height the hospital are the following:
v. Danger signs of pregnancy i. Pregnancy more the 4
4. Determine the stage of labor ii. Previous CS
5. Decide if the woman can safely deliver iii. History of postpartum hemorrhage
6. Give supportive care throughout labor iv. History of medical illness such as heart disease,
7. Monitor and manage labor goiter, tuberculosis, diabetes, severe anemia,
8. Monitor closely after delivery hypertension, bronchial asthma
9. Continue care for at least two hours postpartum. v. Antepartum hemorrhage
vi. Hypertensive disorders of pregnancy and
Home Delivery Eclampsia
 For normal pregnancies attended by licensed vii. Cephalo-pelvic disproportion
health personnel. viii.Placenta previa and abruption placenta
 Trained hilots may be allowed to attend home ix. Multifetal pregnancy
deliveries only in the following circumstances: x. Post term and preterm pregnancies
i. Areas where there are no health personnel on xi. Previous uterine surgery such as myomectomy
maternal care.
ii. When, at the time of delivery, such personnel is Apgar Scoring
not available.  It provides a valuable index for evaluation of the
Qualified for Home Delivery: infant’s at birth.
 Full term  It is based on five signs ranked in order of
 Less than 5 pregnancies importance as follows:
 Cephalic position i. Heart Rate,
 Without existing diseases such as diabetes, ii. Respiratory Effort
bronchial asthma, heart disease, hypertension, iii. Muscle Tone
goiter, tuberculosis, severe anemia. iv. Reflex Irritability and Color.
 No history of complications like hemorrhage during  In general, they made 1 minute of life and 5
previous deliveries. minutes. Each signs is evaluated according to the
 No history of difficult delivery and prolonged labor degree to which it is present and is given a score of
(more than 24 hours for primi and more than 12 0, 1 and 2. The scores of each sign is added
hours for multigravida) together to give a total scores (10 is the maximum).
 Most affected infants may look normal at birth,
Newborn Screening however, they may have large fontanels and
 It is a public health program aimed at the early tongues, big tummies and prolonged yellowish
identification of infants who are affected by certain discoloration of the skin and eyes.
genetic/metabolic/infectious conditions.  Infants are treated with thyroid hormones and it
 Early identification and intervention can lead to continues throughout life. If the disorder is not
significant reduction of morbidity, mortality and detected and hormone replacement is not
associated disabilities in affected infant initiated within two weeks, the baby with CH
Significance: may suffer fro mental and growth retardation
 Most babies with metabolic disorders look “normal” 3. Galactosemia
at birth. By doing NBS, metabolic disorders may be  it is the absence of enzymes necessary for
detected even before clinical signs and symptoms conversion of the milk sugar galactose to
are present. And as a result of this, treatment can glucose.
be given early to prevent consequences of  Affected infants present with difficulty in feeding,
untreated conditions. vomiting and diarrhea, yellowish skin and eyes,
Timing: weakness, white eyes (cat’s eyes) and
 It is ideally done on the 48th-72nd hours of life. bleeding after blood extraction.
However, it may also be done after 24 hours from  Accumulation of excessive galactose in the
birth. body may cause liver damage, brain damage
Procedure: and cataracts. Treatment may include
 A few drops are taken from the baby’s heel, blotted elimination of milk from the diet and use of milk
on a special absorbent filter card and then sent to substitute.
the Newborn Screening Center (NSC). 4. Glucose 6 phosphate dehydrogenase deficiency
 The blood samples for Newborn Screening (NBS) (G6PD deficiency)
may be collected by any of the following:  the body lacks the enzyme called G6PD that
i. Physician may cause hemolytic anemia, when the body
ii. Nurse exposed to oxidative substances found in
iii. medical technologies certain drugs, foods and chemicals.
iv. trained midwife.  Children become pale, with yellow skin and eye,
 The procedure costs P550. The DOH advisory tea colored urine and fast breathing. It may
Committee on Newborn Screening has approved a lead to heart failure.
maximum allowable fee of P50 for the collection of
the sample. Support to Breastfeeding
 Newborn Screening is now included in the  Motivate ,mothers to practice breastfeeding
Philhealth Newborn Care Package. It is widely 1. The Rooming-in and Breastfeeding Act of 1992
available in hospitals, Lyingins, Rural Health Unit,  To encourage, protect and support the practice
Health Centers, and some private clinics. of breastfeeding. It shall create an environment
 If babies are delivered at home, babies may be where the basic physical, emotional and
brought to the nearest institution offering newborn psychological needs of mothers and infants
screening. are fulfilled.
 Results can be claimed from the health facility 2. . Milk Code of 1986
where NBS was availed.  The aim of this code is to contribute to the
 Normal NBS results are available by 7-14 working provision of safe and adequate nutrition for
days from the time samples are received at the infants by the protection and promotion of
NSC. Positive NBS results are relayed to the breastfeeding and by ensuring the proper use
parents immediately by the health facility. of breast milk substitutes and breastmilk
 A negative screen means that the NBS is normal. supplements when these are necessary, on
 A positive screen means that the newborn must be the basis of adequate information and through
brought back to his/her health practitioner for appropriate marketing and distribution.
further testing. Babies with positive results maybe
referred at once to a specialist for confirmatory Family Planning Counseling
testing and further management.  Proper counseling of couples on the importance of
family planning will help them inform on the right
Disorder Detected in Newborn Screening: choices of family planning methods, proper spacing
1. Phenylketonuria of birth and addressing the right number of children.
 it is the inability to metabolize the amino acid Birth spacing of three to five years interval will help
phenylaline, which is a common component completely develop the health of a mother from
such a milk. previous pregnancy and childbirth. The risk of
 Excessive accumulation of phenylalanine in the complications increases after the second birth.
blood causes brain damage. The babies may
look like “albino” with musty odor of the skin, Key Points:
hair, sweat and urine. PKU is treated with a 1. Introduce yourself using the full name.
special low-phenylalanine diet which the 2. Explain what would you like to do and gain her
amount of amino acid is carefully regulated. consent.
2. Congenital Hypothyroidism 3. Ensure the patient is comfortable and warm.
 most common causes of mental retardation. 4. Do not do vaginal or breast exam. alone.
5. All information's are confidential.  Multiple pregnancy , Hydramnios, accidental
hemorrhage, Macrocosmia, Miscalculation.
Approach to Obstetric Patient: 9. Vaginal Discharge
 Investigations  Rupture of membrane, physiological leucorrhoea,
 Examination candidiasis, cervicitis.
 History 10. History of TT/Iron + CA Supplement
Menstrual and Gynecological History-4
History Taking 1. Cycle
 History taking is an ART  1stday of LNMP ( was it conform to the usual in
i. Logical sequence terms of timing, volume, and appearance)
ii. Avoid inadvertent omission of important details.  Regularity
iii. Guide examination.  Length
 Since pregnancy is a “normal” occurrence, the  OCP use.
usual format of the clinical history should be 2. Surgical procedures
modified. 3. Hx of infertility
 History of past pregnancy may alter the outcome of 4. Sexually transmitted diseases
current pregnancy. 5. Uterine anomalies
Personal History-1 Past Obstetric History-5
N→ Name 1. Gravidity, Parity
A→ Age 2. Outcomes
S→ Sex 3. Gestational age: abortion, preterm, term
O → Occupation 4. Delivery
M→ Marital status a) Date
R→ Residency (Address) b) Mode
H→ Special Habits c) Place
G→ Gravidity 5. Infant : sex ,weight, well-being
P→ Parity 6. Complications
Chief Complaint-2
 Chief complaint with duration in chronological order Obstetric History
in patient own words. 1. Drug history and allergy.
1. Amenorrhoea 2. Social
 Expressed in weeks, Calculated from LMP  Cigarette smoking, illegal drug use, domestic
 Diagnosis of pregnancy confirmed violence, psychiatric illness specially in
 Naegele’ s rule (EDD= LMP +9M+7D) postnatal period.
 For accurate estimation: 3. Family Hx
 Menses – regular, average length, sure  Hereditary illness → DM., Hpt., thalassemia,
 No recent use of COC pills. sickle cell disease, hemophilia
 Obstetric calendar (wheel)  Congenital defects eg. neural tube defects,
 First trimester ultrasound scan Down syndrome
 Later ultrasound scan  Twins
2. Symptoms Related to Pregnancy
 Nausea and vomiting, Frequency of Micturation, Obstetric Physical Exam
Constipation, Heaviness in the breast, Rise of Key Points:
temperature. 1. Wash your hands
3. Ankle Edema 2. Introduce yourself using the full name .
 Pressure due to gravid uterus. 3. Explain what would you like to do and gain her
History of Present Illness-3 consent. patient to empty the bladder .
4. Pain Abdomen 4. Do not do vaginal or breast exam. alone.
 Labour pain, abruptio plancetae, pre-eclampsia 5. Ensure the patient is comfortable and warm.
toxemia, degeneration of myoma, acute urinary 6. For abdominal exam: Uncover the patient’s
retension, torsion of ovarian cyst, acute abdomen from the xiphi sternum to the pubic
cholecystitis. hairline, .
5. Backache 7. Abdominal wall relaxation is maximized by (arms
 Increase in body weight, Hyperextension of the along side and hips slightly flexed)
spine, Laxity of joints in pelvis and spine. 8. Advise the mother to indicate if she should feel
6. Vaginal Bleeding weak or nauseous.
 Implantation Hemorrhage 9. All information's are confidential.
 Early pregnancy : Abortion, Ectopic Pregnancy,
V.mole, Procedure:
 Late pregnancy: Placenta Previa, Abruptio Placenta, General
Vasa previa  Local of the general
7. Decreased Fetal MovementT: 10/ 12 hrs i. Breast
 PRIMI: 18 weeks ii. Chest
 MULTI: 16 weeks iii. Abdominal
8. Excessive Enlargement of the Abdomen iv. Heart
v. Back
vi. Lower limbs
Specific
 Local of specific
i. Abdominal
ii. Pelvic
1. General Physical Examination
 Appearance, height, weight, body mass index,
mode, memory, intelligence, orientation (time, place
and persons).
2. Vital Signs
 Pulse, blood pressure, respiration, temperature.
3. Breast Examination 1. First maneuver. One or both hands are placed over
 Chest, heart fundus and the fetal part identified.
 Ophthalmoscopy - hypertensive/diabetic woman. 2. Second maneuver. The palmar surface of one hand
is used to locate the back of the fetus and the other
Blood Pressure hand to feel the irregularities, such as hands and
 Seated, semi-recombent. feet.
 Each visit. 3. Third maneuver. Thumb and third finger are used to
HTN : BP > 140/90 mm Hg on 2 separate grasp presenting part over the pubic symphysis.
occasions 6 H apart and less than 7 days using 5th 4. Fourth maneuver. Both hands are used to outline
Korotkoff sounds . the fetal head.
< 20 wks Chronic HTN .
> 20 wks Gestational HTN.
PET.

Abdominal Exam
 Semi-recumbent position.
 Cover legs with sheet.
3. Auscultation of Fetal Heart
1. Inspection:  Site:
i. Shape of uterus . i. anterior fetal shoulder .
ii. Any asymmetry.
 Time:
iii. Look for fetal movements.
i. 12 wks by sonicade ( US Doppler device)
iv. Look for scars
ii. 24 wks by Pinard steoscope
v. Hernia orifices.
 Duration:
vi. Cutaneous signs of pregnancy → linea nigra, i. Rate
striae gravidarum, striae albicans, umbilicus ii. rhythm over 1 min.
flat or everted, superficial veins
4. Percussion
2. Palpation:
i. Polyhydramnious
i. Superficial palpation
ii. Ballotment
ii. Temperature
iii. fluid thrill
iii. Tenderness
iv. Gardening. Vaginal Examination
Specific palpation:
Pre-requisits:
i. Fundal height → Palpate before 20 wks( fundal
i. Explanation
level) + Measure after 20 wks (Symphysis-
ii. Empty bladder
fundal height)
iii. Dorsal position
ii. Leopold's maneuvers iv. Full asepsis
a) Fundal grip v. Equipment are present
b) Lateral grip
Contradictions:
c) First pelvic grip. PAWLIK’S GRIP
 Placenta praevia
d) Second pelvic grip
 Prelabour rupture of membranes
Symphysis-fundal Height:
Vulva and Perineum:
 Hyper pigmentation
 Look for abnormalities
i. Varicose veins/ hemorrhoids
ii. Warts or herpes
 Cx : Softer, pigmented with ➔ thick , yellowish
mucous secretions
 Uterus : enlarged

Pelvic Assessment
 Check ischial spines if prominent or not
Symphysis-fundal Measurement:
 Diagonal conjugate ➔ distance from lower border
of the symphysis pubis to the sacral promontery
(pelvic inlet)
 Shape of the sacrum
 Side walls of the pelvis i. During the first trimester, occurs at
 Distance between the two sacral promonteries. approximately 0.8 kg (1.5 lb) per month.
ii. During the last two trimesters, 0.4 kg (1 lb) per
Introduction to Maternal and Child Health Nursing week.
 Child, pregnant, family After the first prenatal visit:
 Return appointments are usually scheduled:
Primary Goal: i. every 4 weeks through the 28th weekof
 Promotion and maintenance of optimal family pregnancy.
health. ii. every 2 weeks through the 36th week.
iii. every week until birth.
Scope and Practice on MCHN
1. Pre- conception healthcare Fetal Growth
2. Nursing care on: Typical fundal (top of the uterus)
a) women throughout pregnancy, birth, and measurements are:
postpartum period
b) children from birth through adolescence
c) families in all settings

Philosophy of MCHN:
i. Family-centered
ii. Community-centered
iii. Evidence-based

Pillars of the 2020 National Health Goals


 To increase quality and years of healthy life.
 To eliminate health disparities.

Obstetric Assessment: 1. Over the symphysis pubis at 12 weeks.


1. Assessment of Maternal Weight 2. At the umbilicus at 20 weeks.
 clues that the mother’s nutrition may be 3. At the xiphoid process at 36 weeks.
inadequate for sound fetal growth.
2. Assessment of Bruises How to Locate the Fundus?
 may indicate intimate partner violence that could
have bruised the fetus as well.
3. Assessment of High Blood Pressure
 an elevated blood pressure may be the of
pregnancy, beginning of hypertension which
can restrict fetal growth.

The major causes of serious illness or death during


pregnancy for women today are (According to the
Center for Disease Control and Prevention):
i. hypertension
ii. hemorrhage McDonald’s Rule:
iii. embolism  Used to determine the age of gestation by
iv. infection measuring from the fundus (obtaining the fundal
v. morbid obesity height) to the symphysis pubis.
vi. anesthesia-related  Easy method of determining mid-pregnancy growth.
vii. complications such as intrapartum cardiac arrest.  e.g., in a pregnancy of 24 weeks, the fundal height
should be 24 cm.
Prenatal Care  Becomes inaccurate during the third trimester of
 is a major strategy for helping to reduce pregnancy as fetus is growing more in weight than
complications of pregnancy. in height during this time.
 Ex:preterm or low–birth-weight babies.
1st Prenatal Visit
 obtain baseline data relevant to a woman’s health.
i. health and sexual history.
ii. screening for the risk of teratogen exposure.
 Obtain baseline height/weight and vital sign
measurement.
i. (establish a pregnancy BMI to serve as a
baseline for future comparison)
Pregnancy Weight
 Average woman should gain 25 to 35 lb.
 Weight gain
 Safety
 Informatics

Legal Considerations Specific to Maternal–Child


Nursing Practice
 Informed consent related to fetal well-being.
 Informed consent and legal guardianship for
procedures performed on children.
 Length of time between healthcare incident and
child’s ability to bring lawsuit.
 Identifying and reporting suspected child abuse.
 Concepts of “wrongful birth,” “wrongful life,”
and“wrongful conception”.

Alternative Healthcare Practices Commonly Used


i. Acupuncture
ii. Homeopathy
iii. Therapeutic touch
What if fundal height is much greater than the
iv. Chiropractic car
standard?
v. Herbalism
 Multiple pregnancy
 Miscalculated due date
Statistics Related to the Measurement of Maternal
 Large-for-gestational-age (LGA) infant
and Child Health
 Hydramnios (increased amniotic fluid volume)
1.
 Possibly even Gestational Trophoblastic Disease
i. Birth rate
What if fundal height is less than the standard?
ii. Fertility rate
 Fetus is failing to thrive (e.g., intrauterine growth
iii. Fetal death rate
restriction).
iv. Neonatal death rate
 pregnancy length was miscalculated.
v. Perinatal death rate
 Fetal anomalies.
2.
i. Maternal mortality rate
Maternal and Child Health Nursing Practice
ii. Infant mortality rat
Throughout the Childbearing‒Childrearing
iii. Childhood mortality rate
Continuum:
iv. Childhood morbidity rate
 Provision of preconception health care.
 Provision of nursing care of women throughout
Trends Impacting Maternal and Child Health Nursing:
pregnancy, birth, and postpartum period.
i. Families contain fewer members.
 Provision of nursing care of children from birth
ii. The number of single parents is increasing.
through adolescence.
iii. Ninety percent of women work outside the home;
 Provision of nursing care to families in all settings.
many are the primary wage earner.
iv. The number of homeless women and children is
Framework for Maternal and Child Health
increasing.
Nursing Care
v. Families are becoming more mobile.
1. Phases of health care
vi. Families are more informed.
i. Health promotion-is concerned with prevention
vii. Child and intimate partner violence is increasing.
of disease/illness and promotion of continued
viii. Balancing quality and cost containment in health
good health.
care is an increasing initiative.
ii. Health maintenance-occurs when the client is at
risk for disease/injury.
Leopold’s Maneuver
iii. Health restoration-is concerned with intervention
Fetal lie
when disease/injury has occurred.
 Longitudinal/vertical
iv. Health rehabilitation-to prevent complications
 Transverse
related to existing illness and to allow clients to
 Oblique
achieve an optimum state of wellness.
2. The steps of the nursing process
i. Assessment
ii. Nursing diagnosis
iii. Planning
iv. Implementation
v. Evaluation

Quality & Safety Education for Nurses


(QSEN)
 Patient-Centered Care Fetal attitude
 Teamwork & Collaboration  describes the degree of flexion a fetus assumes
 Evidence-Based during labor or the relation of the fetal parts to each
 Quality Improvement
other.
Fetal Presentation
 denotes the body part that will first contact the
cervix or be born first and is determined by the
combination of fetal lie and the degree of fetal
flexion (attitude).
i. CEPHALIC
a) -Vertex
b) -Brow
c) -Face
d) -Mentum
ii. BREECH Fetal Position
a) -Complete  is the relationship of the presenting part to a
b) -Frank specific quadrant and side of a woman’s pelvis.
c) -Footling
iii. SHOULDER

Position is indicated by an abbreviation of 3 letters:


1st letter:
 Defines whether the landmark is pointing to the
mother’s right (R) or left (L)
2nd letter:
 Middle letter denotes the fetal landmark:
i. In a vertex presentation, the occiput (O) is the
chosen point
ii. In a face presentation, it is the chin (mentum
[M])
iii. In a breech presentation, it is the sacrum (Sa)
iv. In a shoulder presentation, it is the scapula or
the acromion process (A).
3rd letter:
 The last letter defines whether the landmark points
anteriorly (A), posteriorly (P), or transversely (T).

Leopold’s Maneuver
 It is a systematic palpation of the abdomen for
assessing the fetal position and presentation.
Purpose:
i. Determine the number of fetuses.
ii. Identify the PRESENTATION, POSITION,
DEGREE OF DESCENT, ATTITUDE OF THE 11. FOURTH MANEUVER ( PELVIC’S GRIP)
FETUS. i. Turn and face the woman’s feet to confirm the
iii. Identify the point of maximum intensity of the FHT findings of the 3rd maneuver and determine the
in relation to the woman’s abdomen. flexion of the vertex into the pelvis.
Materials Needed: ii. Move fingers of both hands gently down the sides
i. Stethoscope of the abdomen towards the pubis.
ii. Pillow
iii. Towel Palpate for the Cephalic Prominence (Vertex):
Procedure:  Prominence on the same side as the small parts
1. Wash hands suggests that the head is flexed (optimum).
R: To deter the spread of microorganism  Prominence on the same side as the back suggests
2. Explain the procedure to the patient. that the head is extended.
R: To gain patient’s cooperation.
3. Provide patient’s privacy
R: To ensure patient’s comfort
4. Let the patient empty her bladder before the
procedure
R: A distended bladder may slightly deviate the uterus.
5. Position the patient on supine with one pillow under
her head, knees slightly flexed and expose the abdomen
R: To facilitate accurate assessment and providing
comfort.
A flexed knees relieve tension of abdominal
Musculature.
6. Place rolled towel under patient’s right hip.
R: To shift uterus away from large blood vessels and
thus prevent supine hypotensive syndrome.
12. Locate again the back of the fetus and place the
7. If right handed, stand at woman’s right, facing the stethoscope over it and listen to the Fetal
patient. Heart Tone for one full minute.
R: To easily grasp the symphysis pubis to be done in the
 Normal FHT: 120-160 beats per minute.
3rd maneuver
13. Note the LOCATION, RATE and CHARACTER of
8. FIRST MANEUVER (FUNDAL GRIP)
the FHT
i. Determine whether fetal head or breech (Buttocks)
 In CEPHALIC PRESENTATION, FHT can be heard
is in the fundus in either left or right lower quadrant of the abdomen.
ii. Face the patient and palpate the uterine fundus to  In BREECH, FHT can be heard ABOVE the level of
determine what part of the fetus lies in the upper
the umbilicus, either left or right.
part of the fundus
14. Make the patient comfortable.
iii. HEAD feels HARD AND ROUND, freely movable
R: To ease and promote relaxation.
and ballotable
15. Document the observation made.
iv. BREECH feels SOFTER and NODULAR  The fetal findings
9. SECOND MANEUVER (UMBILICAL GRIP) i. FHT
i. Determines position of the fetal back
ii. Presentation
ii. Palpate in a downward direction on the sides of the
iii. Position
abdomen by applying gentle but deep pressure to
iv. Attitude
determine the position of the fetal extremities, fetal
v. Whether engaged or floating
back and anterior shoulder. R: To promote database for future.
iii. Palpate with one hand steady on one side, the
other hand exploring on the other side.
Basic Fetal Monitoring
 Electrical Fetal Monitoring
[IF FETAL BACK, A LONG CONTINUOUS
 Method of assessing fetal status before and during
STRUCTURE WILL BE FELT. IF EXTREMITIES, IT
labor.
WILL FEEL NODULAR REFLECTING THE PORTION
 Information is recorded on graph paper
OF FETAL EXTREMETIES.]
 Information is permanent part of the maternal
medical record.
10. THIRD MANEUVER (PAWLICK’S
 Information is retrievable for litigation.
GRIP)
i. Determine the presenting part and if engagement
Importance:
has occurred.
 To provide insight that may affect fetal outcomes.
ii. Place one hand over the symphysis pubis and
grasp the lower uterine segment between the
Normal Assessment Findings:
thumb and fingers to feel the presenting part.
1. FHR between 110-160 in gestations 32-40+ weeks.
a) Rates slightly above 160 are normal in
[THE HEAD IS AT THE INLET OR IN PELVIS, IN ZERO
gestations less than 32 weeks.
(0) STATION IF PRESENTING PART IS STILL
2. Regular rhythm
MOVABLE, IT IS NOT ENGAGED.]
3. Increases in the FHR associated with fetal  Small squares represent 10 bpm increases as
movement that return to original rate range. well as 10 seconds duration.
2. Lower Graph
Electronic Fetal Monitoring Clarification:  Records contraction data.
 Information for students is for educational purposes  Small squares represent 10 second duration or
only. 10 mmHg intensity.
 Students should not assume any responsibility for [Dark line to dark line represents one minute of time.]
interpretation of fetal monitor tracings.
 It takes months to years of experience to be Baseline Fetal Heart Rate (FHR):
prepared to interpret fetal monitor tracings.  Normal baseline FHR in a term fetus 37 completed
weeks or more is 110-160 BPM.
Methods of Electronic Fetal Monitoring: i. Determination of the baseline FHR is done
1. External between contractions.
 Noninvasive method. ii. Baseline is rounded in increments of 5 bpm
 Utilizes an ultrasonic transducer to monitor the example; if the FHR is running 125-135 then
fetal heart. the baseline FHR should be documented as
 Utilizes the tocodynamometer (toco) to monitor 130.
uterine contraction pattern.

Fetal Heart Rate (FHR) Variability


2. Internal  Normal changes and fluctuations in the FHR over
 Invasive time.
 FHR is monitored via a fetal scalp electrode.  Best assessed between contractions.
 Uterine activity is monitored by an intrauterine  Considered to be the best indicator of fetal well-
pressure catheter (IUPC). being.
 Variability can be influenced by hypoxic events,
maternal hemodynamic issues, drugs, etc.
Ex:
Absent: Not detectable from baseline.
Minimal: Less than 5 bpm from baseline.
i. May occur with:
a) normal fetal sleep patterns
b) mother has received analgesia for pain
Moderate : 6-25 bpm from baseline (optimal pattern).
Marked: More than 25 bpm from baseline.
3. Combination
 Combination of external and internal fetal How do Uterine Contractions Affect Fetal Heart Rate:
monitoring is a common practice.  Can affect FHR by increasing or decreasing the
rate in association with any given contraction.
Advantages and Disadvantages of Internal Fetal  3 primary mechanisms by which UCs can cause a
Monitoring: decrease in FHR
1. Advantages i. Fetal head
i. Patient can move without much interference in ii. Umbilical cord
data transmission. iii. Uterine myometrial vessels
ii. More accurate measurement of data.
iii. Data less likely to be affected by artifact.
2. Disadvantages
i. Invasive
ii. Membranes have to be ruptured and cervix
dilated.
iii. Application requires more skill.
iv. Procedure is uncomfortable for the mother.
v. Risk of trauma and infection for mother and
fetus.
Periodic and Episodic FHR Characteristics:
Components of the Fetal Monitor Paper Tracing: 1. Periodic
1. Upper Graph  Refers to changes in the FHR that occur with or
 Records FHR data. in relationship to contractions.
Ex:
Variable Acceleration: Results from some type of cord  Commonly associated with uterine hyperstimulation.
compression.  Can also occur without any uterine activity.
 Nuchal cord
 True knot
 Decreased amniotic fluid

FHR Accelerations
 the increase from the fetal heart baseline

2. Episodic
 Refers to changes in the FHR that occur
independent of contractions.

Severe Variable Deceleration:


1. Early Deceleration
 Occur as a result of vagal stimulation to the fetal
head during contractions which push the fetal head
toward the pelvis.
Sinusoidal Pattern
 Persistent wave variation of the baseline only seen
in about 2% of patients.
 Related to severe fetal anemia, hypoxia, or acidosis.

2. Late Deceleration
 Occur in response to utero-placental insufficiency.
 Blood flow to the fetus is compromised and there is
less oxygen available to the fetus).

Uterine Activity Assessment


 Periodic tightening and relaxing of the uterine
muscle.
 Pituitary gland is triggered to release a hormone
called oxytocin that stimulates the uterine tightening.
 Difference in Braxton Hicks contractions and true
labor is the strength of the contractions and the
changes in the cervix.

3. Late Deceleration with Absent Variability Characteristics of Contractions:


 Note the smoothness of the FHR pattern. 1. Frequency: How often they occur? They are timed
 Decreased FHR caused from the beginning of a contraction to the beginning of
 by utero-placental insufficiency. the next contraction.
 Compromised blood flow to fetus. 2. Regularity: Is the pattern rhythmic?
3. Duration: From beginning to end - How long does
each contraction last?
4. Intensity: By palpation mild, moderate, or strong.
5. By IUPC intensity in mmHg
6. Subjectively: Patient description

4. Prolonged Deceleration
 Deceleration of the FHR from the baseline lasting
more than 2 minutes but less than 10 minutes.
 No explanation for why these occur.
ANSWER IS 1
 Accelerations are transient increases in the fetal
heart rate that often accompany contractions or are
caused by fetal movement.
 Episodic accelerations are thought to be a sign of
fetal-well being and adequate oxygen reserve.

A NURSE IS ADMITTING A PREGNANT CLIENT TO


THE LABOR ROOM AND ATTACHES AN EXTERNAL
ELECTRONIC FETAL MONITOR TO THE CLIENT’S
Segment of Contractions: ABDOMEN. AFTER ATTACHMENT OF THE MONITOR,
1. Increment: Beginning, building of pressure THE INITIAL NURSING ASSESSMENT IS WHICH OF
2. Acme: Most intense part of the contraction THE FOLLOWING?
3. Decrement: Diminishing of the contraction 1. Identifying the types of accelerations
4. Rest: Period of time between contractions 2. Assessing the baseline fetal heart rate
3. Determining the frequency of the contractions
4. Determining the intensity of the contractions
ANSWER IS 2
 Assessing the baseline fetal heart rate is important
so that abnormal variations of the baseline rate will
be identified if they occur.
 Options 1 and 3 are important to assess, but not as
the first priority.

A NURSE IS MONITORING A CLIENT IN LABOR.


Assessment of Contractions: THE NURSE SUSPECTS UMBILICAL CORD
 Palpation: Use the fingertips to palpate the fundus COMPRESSION IF WHICH OF THE FOLLOWING IS
of the uterus NOTED ON THE EXTERNAL MONITOR TRACING
a) Mild: Uterus can be indented with gentle DURING A CONTRACTION?
pressure at peak of contraction. 1. Early decelerations
b) Moderate: Uterus can be indented with firm 2. Variable decelerations
pressure at peak of contraction. 3. Late decelerations
c) Strong: Uterus feels firm and cannot be 4. Short-term variability
indented during peak of contraction. ANSWER IS 2
 Variable decelerations occur if the umbilical cord
EXERCISES: becomes compressed, thus reducing blood flow
between the placenta and the fetus.
VARIABLE DECELERATIONS IN FHR DURING  Early decelerations result from pressure on the fetal
LABOR ARE SEVERE DIPS OCCURRING AT THE head during a contraction.
PEAK OF CONTRACTION. THIS FHR PROBLEM IS  Late decelerations are an suggests utero-placental
ASSOCIATED WITH WHICH ONE OF THE insufficiency during a contraction.
FOLLOWING CONDITIONS?  Short-term variability refers to the beat-to-beat
1. Utero-placental insufficiency range in the fetal heart rate.
2. Fetal head compression
3. Uterine insufficiency THE PHYSICIAN ASKS THE NURSE THE
4. Pressure on the umbilical cord FREQUENCY OF A LABORING CLIENT’S
ANSWER IS 4 CONTRACTIONS. THE NURSE ASSESSES THE
 These decelerations are common during labor. CLIENT’S CONTRACTIONS BY TIMING FROM THE
 The FHR drops during the contraction resulting BEGINNING OF ONE CONTRACTION:
from stimulation from chemoreceptors and 1. Until the time it is completely over
baroreceptors as the cord is compressed. 2. To the end of a second contraction
 The nurse should recognize these readings on the 3. To the beginning of the next contraction
fetal monitor as normal. 4. Until the time that the uterus becomes very firm
ANSWER IS 3
[A NURSE IS CARING FOR A CLIENT IN LABOR AND  This is the way to determine the frequency of the
IS MONITORING THE FHR PATTERNS. THE NURSE contractions.
NOTES THE PRESENCE OF EPISODIC
ACCELERATIONS ON THE ELECTRONIC FETAL WHEN MONITORING THE FHR OF A CLIENT IN
MONITOR TRACING. WHICH OF THE FOLLOWING LABOR, THE NURSE IDENTIFIES AN ELEVATION OF
ACTIONS IS MOST APPROPRIATE? 15 BEATS ABOVE THE BASELINE RATE OF 135
1. Document the findings and tell the mother that the BEATS PER MINUTE LASTING FOR 15 SECONDS.
monitor indicates fetal well-being THIS SHOULD BE DOCUMENTED AS:
2. Take the mothers vital signs and tell the mother that 1. An acceleration
bed rest is required to conserve oxygen. 2. An early elevation
3. Notify the physician of the findings. 3. A sonographic motion
4. Reposition the mother and check the monitor for 4. A tachycardic heart rate
changes in the fetal tracing
ANSWER IS 1 a) There is a decrease in maternal mortality and
 An acceleration is an abrupt elevation above the morbidity owing to obstructed labor. (Third
baseline of 15 beats per minute for 15 seconds; if Report on Confidential Inquiries into Maternal
the acceleration persists for more than 10 minutes it Deaths in South Africa, 2002-2001, 2006:7)
is considered a change in baseline rate.
 A tachycardic FHR is above 160 beats per minute. Components of the Partograph:
Part 1
WHICH OF THE FOLLOWING FINDINGS MEETS THE  Progress of labor
CRITERIA OF A REASSURING FHR PATTERN? i. Cervicogram area
1. FHR does not change as a result of fetal activity ii. Each square is 1cm cervical dilation
2. Average baseline rate ranges between 100 - 140 iii. Active phase:
BPM a) Multipara = 1.5cm/hr
3. Mild late deceleration patterns occur with some b) Primipara = 1 cm/hr
contractions iv. Alert line
4. Variability averages between 6 - 10 BPM ANSWER a) Drawn from 4-10 cm
IS 4 b) Represents rate of dilation
 Variability indicates a well oxygenated fetus with a c) Slow rate of dilation indicates delay in
functioning autonomic nervous system. labor.
 FHR should accelerate with fetal movement. v. Action line
 Baseline range for the FHR is 120 to 160 beats per a) Drawn 4 hrs to the right of the alert line
minute. and parallel to it.
 Late deceleration patterns are never reassuring, b) Referred to as critical line.
though early and mild variable decelerations are c) If cervical dilation crosses the line:
expected, reassuring findings. i. Critical assessment of the cause of the
delay.
-------------------------------------------------------------------- ii. Decide on appropriate management
should be undertaken.
PARTOGRAPH
Principle:
 A tool for prevention of fetal and maternal
complications during labor.
 Record clinical observations accurately on the
partograph.

Partograph
 Came from a Greek word that means “labor curve”.
 Originally designed and used by Prof. R.H. Philpott
of Zimbabwe in 1972.
 Later modified and simplified by WHO.
 A simple, inexpensive graphical record which gives
continuous pictorial overview of the progress of all
observations made of a woman in labor.
 A vital tool to identify complications in childbirth in a
timely manner.

Reasons for Using the Partograph:


1. It is an assessment tool that is easy to use.
a) Enable nurses to see progress of labor at a
glance on one sheet of paper.
b) Replaces lengthy descriptions.
2. Various studies have indicated advantages and
disadvantages of using the partograph:
a) Reduces the number of prolonged labor,
labors requiring oxytocin augmentation and CS.
b) Fahdly and Chonsuvivavatwong, 2005:301
c) WHO, 1991:1
d) Odberg, Petterson et al., 200:83 Plotting the Progress of Labor
3. Assists in early decision on transfer, augmentation  Plot only the cervical dilation using the symbol “X”.
or termination of labor.  Start when woman is in active labor (4cm or more)
a) Can be used by peripheral personnel in their and is contracting adequately (3-4 contractions in
referral decisions (WHO, 1991:1) 10 mins).
4. Increases quality and regularity of all observations If plotting passes alert line:
on the fetus and mother during labor and aids in  Reassess woman and consider criteria for referral.
early recognition of problems with either of them.  Alert transport services.
5. Recognize maternal or fetal problems as early as  Encourage woman to empty bladder.
possible.
 Encourage upright position and walking if woman
wishes.
 Monitor intensively.
 If referral takes a long time, refer immediately. DO
NOT WAIT TO CROSS ACTION LINE.
If plotting reaches the action line:
 Refer urgently to hospital unless birth is imminent.
 If woman is admitted in Latent Phase (less than
4cm dilated), record only other findings (BP, FHT,
etc).
 If she remains in latent phase for next 8 hrs(labor is Part 2
prolonged), transfer her to hospital.  Assessment of maternal condition
i. Monitor q4h:
a) Drugs
b) Pulse
c) Blood pressure
d) Temperature
e) Urine volume
Part 3
 Assessment of fetal condition
i. Regular monitoring of:
a) FHR
b) Membranes and amniotic fluid.
 “I” - Intact membranes
If ruptured, note color of amniotic fluid:
 “C” - Clear
 “A” - Absent
 “M” - Meconium-stained
 “B” - Blood-stained
Part 4
 Outcome of labor
i. Based on observations of part 1, 2, and 3 and
after delivery.
-------------------------------------------------------------------

Obstetric History and Examination


Health Assessment:
 A plan of care that identifies the specific needs,
medical problems, and how those needs will be
addressed.
 Health history
 Physical examination
 Laboratory & diagnostics test
Objectives:
 Patient demographics
 Current pregnancy details and complaints
 Past obstetric history
 Past gynecological history
 Past medical and surgical history
 Drug history and allergies
 Family history
 Social history
 Systemic review

Patient Demographics
i. Name
ii. Age
iii. Occupation
iv. Relationship status
v. Booking status
vi. Gravidity
vii. Parity
viii. Last Menstrual Period (LMP)
ix. Estimates Date of Delivery (EDD)
a) Naegele’s Rule - add 1 year and 7 days to
the LMP and subtract 3 months).
 Cervical smear history (last smear, when, where,
Gravidity and Parity what was the result, awareness and follow up
Gravida plans).
 Number of pregnancies a woman has had G4.  Methods of contraception
Para-G4P3  Difficulties in conceiving?
 Number of viable pregnancies regardless of
number and outcome. Past Medical and Surgical History
FPAL  Any illness in childhood or adult life (DM, HTN,
 Full term, preterm, abortion, living. Hepatitis, Psychiatric illnesses, epilepsy).
TPAL  Previous hospitalizations (when, where, why, how
 Term, preterm, abortion, living. long).
GTPALM  Past surgery: Any past surgical procedures,
 Gravida, term, preterm, abortion, living, particularly any abdominal or gynaecological
multiple operations as well as any associated complications
 G4T1P0A1L2M1 or reaction to anaesthesia.
 G5T1P1A1L2M1
 G6T2P1A1L4M2 Drug History
 Current medications before and after conception
History of Present Illness (prescribed, over the counter, herbal).
1st Trimester  Name
 Planned/unplanned.  Dosage
 Method of confirmation of pregnancy.  Purpose
 General health (tiredness, malaise and other non  Route
specific symptoms).  Frequency
 Booking (when, where, how many visits).  Pregnancy related medication (folic acid, iron,
 Early booking investigations and result (ogtt Hb antiemetic).
electrophoresis, Blood group and Rh, VDRL, HIV) .  Allergies (what exactly happened).
 History of vaginal discharge, vaginal bleeding,  Don’t forget vitamins and nutritional supplements.
urinary problems and flu like symptoms.
 Imaging (crown rump length usually between 9-14 Family History
weeks).  Major illness in the immediate family members (DM,
2nd Trimester HTN, carcinoma of breast, ovary, colon,
 History of fetal movements. endometrium).
 Symptoms of anemia, miscarriage, ectopic  Family history of preeclampsia, eclampsia, DM.
pregnancy (classic triad- amennorhea, abdominal  Genetic disorders: sickle cell disease, cystic fibrosis,
pain, vaginal bleeding), vaginal discharge, UTI. chromosomal anomalies.
 Symptoms of preterm labour, diabetes.  Previously affected pregnancies.
 Imaging (head circumference).  History of twin.
 Anomaly scanning? (when, where, why).
 Blood pressure check up. Social History
 Changes in weight.  Personal status (smoking and alcohol: amount,
3rd Trimester duration and type).
 Any medication due to HPN, DM, CONVULSION.  Occupation
 Any labour pains, vaginal discharge, bleeding,  Educational background
urinary problems.  Socioeconomic status (home conditions, water
 Hospital stays? supply, sanitation).
 Any plans of delivery?  Financial earning of support system.
 How many people live in the household.
Past Obstetric History  Domestic violence screening.
 Details of all previous pregnancies (including  Plans for breastfeeding.
miscarriages and terminations).
 Length of gestation. Examination Intro:
 Date and place of delivery. 1. Introduce yourself and gain consent.
 Onset of labour (including details of induction of 2. Explain the need and nature of the proposed exam.
labour). 3. Respect patient’s privacy at all times.
 Mode of delivery 4. Patient should be covered at all times and relevant
 Sex and birth weight parts of her anatomy only exposed.
 Fetal and neonatal life 5. Ensure room is well lit and comfortable.
 Clear details of complications or adverse outcomes 6. Patient should empty bladder before exam.
(shoulder dystocia, post partum heamorrhage, still 7. Should lie supine with pillow under her head and
birth). arms at the side.
8. Ask for any tenderness before palpation.
Gynecological History
 Age of menarche General
 Regular/irregular cycles  Measure BMI (Body Mass Index) [weight
 LMP, duration of menses, cycle length (kg)/height (m)2].
 Pregnancy complications are increased with BMI d) Distended Superficial Veins (increased IVC
<18.5 and >25. pressure due to gravid uterus).
 Measure vitals (BP, Temperature, Pulse, Resp rate). Linea Nigra
 Blood glucose levels  Dark vertical line appearing on the abdomen from
the pubis to above the umbilicus during pregnancy
Systematic Review due to increase melanocyte stimulating hormone
 General Appearance made by the placenta.
 CVS (chest pain, SOB, palpitations, orthopnea)
 GI
 Genital (pain, discomfort, itch, discharge, bleeding)
 Urinary (frequency, urgency, dysuria, nocturia,
incontinence, character of urine)
 CNS
 MSK (pain, swelling, weakness, gait)

Physiologic Changes in Pregnancy


Reproductive System Changes
1. UTERINE CHANGES Striae Gravidarum
a) Length grows: 6.5 cm to 32 cm  Specific scarring of the skin due to sudden weight
b) Depth increase : 2.5 cm to 22 cm gain during pregnancy. Caused by tearing of the
c) Width expands : 4 cm to 24 cm dermis and results in atrophy.
d) Weight increase : 50 g to 1000g
e) Volume: uterus 1000ml, fetus 7 lbs, amniotic
fluid 1000ml.
Hegar’s sign
 extreme softening of the lower uterine segment.
Balottement
 fetus can be felt to bounce and rise in the amniotic
fluid, 16th to 20th weeks.
Braxtons Hicks Contractions
 practice contractions.
Amenorrhea
 absence of menstrual flow. Laboratory Assessment:
2. CERVICAL CHANGES  Urinalysis: glucose, protein, leukocytes.
Cervix  Blood serum: CBC, blood type
 more vascular and edematous.  Ultrasonograhy
Operculum  Papanicolaou smear (Pap Smear)
 mucus plug, form to seal out bacteria and help.  Glucose screen- 24th & 28th wk AOG
prevent infection  Hemoglucose Tolerance Test (HGTT)/OGTT
Goodell’s Sign
 violet hue, darken cervix, increased vascularity. -----------------------------------------
3. VAGINAL CHANGES
 Slight white vaginal discharge, muscle fiber loosen,
increase epithelial cells . Essential Immediate Newborn Care
Chadwick’s sign
 change in color to deep violet in vaginal wall  Newborn Care protocol developed by the
pH level. department of health.
 4-5, acidic.  Adapted from international and evidence based
4. OVARIAN CHANGES standards set by the WHO.
 Ovulation stops  Directly address the MDG to REDUCE CHILD
 No FSH, LH MORTALITY.
5. CHANGES IN BREASTS  Immediate nursing care essential to the needs of
 Feeling of fullness, tingling, tenderness, due to high neonate provided at the time of birth.
estrogen level.
 Nipple and areola; darken. Administrative Order No. 2009-0025
 Adopting New Policies and Protocol on Essential
Inspection: Newborn Care.
1. Distention  Issued on December 7, 2009
2. Fetal movements.  Outlines specific policies & principles to follow for
3. Scars (especially lower segment all health care providers involved in newborn health
transverse/longitudinal in the event of previous C care.
section). Four steps to save newborn lives
4. Skin changes 1. Immediate and thorough drying of the newborn.
a) Linea nigra 2. Early skin-to-skin contact between mother &
b) Striae Gravidarum newborn.
c) Striae Albicans 3. Properly timed cord clamping and cutting.
4. Non-separation of newborn and mother for early
breastfeeding.

Time Band: Preparing for Delivery


1. Introduce yourself.
R: Gain rapport and cooperation.
2. Review with the mother what care to expect for
herself and her baby.
R: Lessen anxiety of mother, learn info.
3. Ensure privacy and safety.
R: For security, confidentiality. Interpretation of Result
4. Checks delivery area is draft-free and adjusts 0–3
temperature between 25-28C.  in serious danger, needs immediate resuscitation.
R: Providing warmth, prevent hypothermia. 4 -6
5. Perform hand washing.  condition is guarded.
R: Deters the spread of microorganisms. 7 – 10
6. Prepares all needed materials, supplies, equipment.  is in best possible health.
Maintains sterility.
7. Sterile gloves, sterile towel, sterile set containing: Time Band: 1-3 Minutes: Early Skin-to-Skin Contact
sterile umbilical clamp, sterile cutting scissors, 3- and Timely Cord Clamping
operating sponge) 1. Remove wet towel to start skin-to-skin contact.
8. Bonnet, name tag, opthalmic ointment, vitamin K, R: Maintain baby dry and start bonding between mother
Hepa-B vaccine, BCG Vaccine, 1- tuberculin and baby.
syringe, 2- 3 cc syringe, tape measure, cotton ball, 2. Cover the baby with sterile towel and cover head
warmer, crib. with bonnet.
R: To prevent excessive heat loss. Hypothermia
Time Band: At Perineal Bulging with Presenting Part consume 200 cal/kg/min.
Visible 3. Continue skin-to-skin contact on mother’s abdomen
1. Perform medical hand washing. or chest.
2. Put on sterile gloves. R: To create bond, allow colonization of good bacteria,
protect from hypoglycemia.
TO BE DONE BY THE HANDLE
4. Palpates the umbilical cord to check for pulsation.
5. Clamps and cut the cord using the plastic clamp2
cm from the base after pulsation stop or within 1-3
minutes after birth.
R: To prevent anemia and intravascular bleeding.

Time Band: Within 90 Minutes


1. Do not separate the baby from the mother for at
least 60 minutes, unless in respiratory distress or
Time Band: At Birth to 30 Seconds: Immediate with maternal emergency.
Drying R: Increase the success of breast feeding.
1. Deliver the baby onto the sterile towel draped over 2. Encourage breastfeeding when the baby shows
the mother’s abdomen. feeding cues.
R: Prevent contamination and promote early bond. R: Opening of mouth, tonguing, licking, rooting
2. Calls out time of birth and sex of the baby. Emphasis the importance of breastfeeding. Stimulate
R: Acknowledge the time of birth and gender uterine contraction, prevent post partum bleeding.
identification. Immunity and nutrition for the baby.
3. Immediately dry the baby within 5 seconds after 3. Do eye care before 1 hour, administer erythromycin
birth. or tetracycline ointment. Apply from inner to outer.
R: Evaporation – loss of heat through conversion of a R: To Protect from eye infection, OPTHALMIA
liquid to a vapour. NEONATORUM.
4. Wipes eyes, face, head, trunk, back, arms and legs 4. Monitor the baby every 15 minutes.
thoroughly. R: Vital signs are good indicator of baby’s adaptation
R: Stimulate the baby to breath. and survival. HR, RR, Temp , Oxygen saturation,
5. Checks breathing while drying. APGAR. Postpone bathing until after 24 hours. Prevent
R: Observe early signs of respiratory distress such as hypothermia and infection.
gasping, grunting, weak cry.
Time Band: Within 90 Minutes to 6 Hours: Newborn
Care
1. Examine the baby.
R: Assess wellness and any possible congenital
anomaly.
2. Put an identification tag around the ankle
R: Prevent misidentification and medical error
3. Weigh the baby and record Economical
R: Assess nutritional status, and determine SGA or LGA. Antibodies- greater immunity to infections
4. Performs anthropometric measurements Stool inoffensive – never constipated
R: Head circumference is slightly larger than chest, The varying composition of breastmilk keeps pace with
5. Injects Vitamin K the infant’s individual growth and changing nutritional
R: 0.1ml IM , Left Vastus Lateralis needs.
6. Injects hepatitis B vaccine Fresh milk- never goes sour in the breast
R: 0.5ml IM, Right Vastus lateralis Enhances development and intelligence
7. Injects BCG vaccine Easy once established
R: 0.1ml ID, Right Deltoid Develops a special relationship and bonding with her
child.
Vital Signs Immediately available
 Checking the Vital signs of newborn (baseline data) Nursing promotes facial structure development,
Temperature = 36.5 – 37.5 C straighter teeth and enhances vision.
Heart Rate = 120 – 160 bpm Gastroenteritis greatly reduced
Respiration = 30 – 60 cycles per min
Elements Necessary in Normal Breastfeeding
Proper Identification Process:
 Must be done in the delivery room. 1. A BREAST that produces and releases milk
Identification bands 2. A BABY who is liable to remove the milk from the
 a plastic bracelet attached to newborn’s wrist or breast with effective suckling.
ankle with permanent locks that require cutting to  The manner in which the baby is attached at the
be removed: breast will determine how successful these 2
 e.g. Name: Bb. Boy Ramos B-date: 6/22/12 Weight: elements come together.
6.5 lbs. Time: 2:36 pm Pedia: Dr. J. Tan
Types of Breasts:
Weight
 Weigh the baby.
 Normal weight = 2.5 – 3.7 kg
6-9 lbs
2,500- 4,000 gms

Anthropometric Measurement
 Head Circumference = 33 – 35 cm.
 Chest Circumference = 31 – 33 cm
 Abdominal Circumference = 31 – 33cm
 Length = 46 – 54 cm
Breastmilk Production
Documentation:  Hormones or chemical messengers in the blood.
 (Recording of data gathered)  During pregnancy, hormones help breasts:
 weight  To develop and grow
 Vital signs  To start to make colostrum
 Anthropometric measurements  After delivery, hormones of pregnancy decreases.
 Vit.K, Hepa B vaccine, BCG vaccine  Prolactin and Oxytocin are important to help in the
 Apgar Score production and flow of milk.
 any unusualities  Two hormones related closely to breastfeeding are
 Prolactin - causes your alveoli to take nutrients
------------------------------------------------------------------- (proteins, sugars) from your blood supply and
turn them into breastmilk. Can make mother
feel sleepy and relax. Level is HIGH 2 hours
Breastfeeding after birth and at night.
 Is the feeding of babies and young children with  Oxytocin - causes the cells around the alveoli to
milk from a woman’s breast. It is recommended to contract and eject your milk down the milk
begin breastfeeding within the first hour of life and ducts. This passing of the milk down the ducts
to allow it as often and as much as the baby wants. is called the “letdown” (milk ejection) reflex.
Breastmilk
 Is the milk produce by the breast (or mammary Signs of Oxytocin Reflex:
glands) of a human female for her infant offspring. 1. Painful uterine contraction, with rush of blood
 Milk is the primary source of nutrition for newborns (sometimes).
before they are able to eat and digest other foods; 2. A sudden thirst
older infants and toddlers may continue to be 3. Milk Spray from the breast/leaking breast which is
breastfeed, either exclusively or in combination with NOT being suckled.
other foods from around six months of age when 4. Feels a squeezing sensation in the breast.
solid foods may be introduced.

Best for mothers and babies


Reduces risk of childhood obesity
Interference with let-down:
1. A variety of factors may interfere with let-down:
2. Emotions such as embarrassment, anger, irritation,
fear or resentment
3. Fatigue
4. Poor suckling from improper positioning
5. Not enough time baby is actively nursing
6. Stress
7. Negative remarks from relatives or friends
8. Pain in your breasts or uterus (i.e., sore nipples or
afterbirth pains) Artificial Teats/ Pacifiers
9. Breast engorgement in the first few days  Cause difficulties because it has a different mouth
10. Milk contains a small whey protein called Feedback action.
Inhibitor of  May come to prefer the artificial teats.
Lactation (FIL)  May reduce the suckling time at the breast-less
 the role of FIL appears to be to slow milk synthesis stimulation-less production.
when the breast is full. Thus milk production slows
when milk accumulates in the breast (and more FIL Breast Care:
is present).  Washing the breast once a day as part of general
body hygiene is sufficient. Clean with water only.
Role of Baby in Milk Transfer:  Avoid using:
 The Baby’s suckling controls the:  Soap, Vaseline, oils, lotions
 Prolactin production
 Oxytocin reflex Woman who need Extra Attention:
 Removal of the inhibitor within the breast 1. Had difficulty breastfeeding a previous baby
2. Must spend time away from baby
3. Has family difficulty
4. Is depressed
5. Is isolated, without social support
6. Is young, single mother
7. Intention to leave the baby for adoption
8. Previous breast surgery or trauma
9. Has chronic illness or needs medication
10. Baby at high-risk of needing special care after birth
11. Is tested and known to be HIV (+)

Types and Composition of Human Breast Milk: Types of Nipples:


Breast Engorgement Treatment of Mastitis:
 Frequent emptying of breast
 Check attachment
 Help milk flow
 Gentle massage
 Warm compress
 Check clothing
 Rest with baby

Drug treatment for Mastitis:


 Anti-inflammatory
 Ibuprofen (mild analgesic)
 Antibiotic
Practices to avoid engorgement  if indicated 10-14 days
 Skin to skin, initiate BF within 1 hour  Generally oral antibiotic
 Offer help early show how to express milk  (erythromycin, flucloxacillin, dicloxacillin,
 Keep together 24 hours a day amoxicillin, cephalexin).
 Breastfeed on demand
 No pacifiers, teats Sore Nipples
What causes nipple soreness?
Why Help relieve engorgement: i. Shallow latch
 Relieve discomfort ii. Breast pump trauma
 Prevent further complications iii. Candida infection (also called yeast, monilla or
 Ensure continuous milk production thrush)
 Enable baby to receive breastmilk iv. Tongue-tie
v. Milk bleb
How to relieve engorgement: vi. Nipple blister
 Check attachment vii. Vasospasm
 Express between feeds
 Encourage frequent feeds Management of Sore Nipples:
 Apply warm compress  Reassurance
 Massage  Treat cause/s:
 Help mother to be comfortable  Improve attachment & positioning
 Provide supportive atmosphere  Treat source of irritation Candida? Short
 Cold compress frenulum?
 Comfort measure
Blocked Duct and Mastits
 A plugged (or blocked) duct is an area of the breast What does NOT help sore nipple?
where milk flow is obstructed. The nipple pore may  DO NOT stop breastfeeding
be blocked, or the obstruction may be further back  DO NOT limit breastfeeding
in the ductal system. A plugged duct usually comes  DO NOT apply any substance
on gradually and affects only one breast.  DO NOT use nipple shield
Mastitis
 is an inflammation of the breast that can be caused
by obstruction, infection and/or allergy. Mastitis is
most common in the first 2-3 weeks, but can occur
at any stage of lactation.
 Mastitis may come on abruptly, and usually affects
only one breast.

Breastfeeding Position:
 Advertising, promotion, and other marketing
materials that are not approved by the IAC.

Republic Act 7600


 “The Rooming-In and Breast-feeding Act of 1992”
 An act providing incentives to all government and
private health institutions with rooming-in and
breastfeeding practices and for other purposes.

Republic Act No. 10028


 “Expanded Breastfeeding Promotion Act of 2009”
 An act expanding the promotion of breastfeeding an
ending for the purpose Republic Act No. 7600,
other wise known as “An act providing incentives to
all government and private health institutions with
rooming-in and breastfeeding practices and for
other purposes.”

Lactation Stations
 Sec. 11. Establishment of Lactation Stations. – It is
hereby mandated that all health and non-health
facilities, establishments or institutions shall
What is the Milk Code? establish location stations. The lactation stations
 E.O. 51 (Executive Order 51) commonly referred to shall be adequately provided with the necessary
as, “The Milk Code”, is a law that ensures safe and equipment and facilities, such as: lavatory for hand-
adequate nutrition for infants through the promotion washing, unless there is an easily-accessible
of breastfeeding and the regulation of promotion, lavatory nearby; refrigeration or appropriate cooling
distribution, selling, advertising, product public facilities for storing expressed breastmilk; electrical
relations, and information services artificial milk outlets for breast pumps; a small table; comfortable
formulas and other covered products. seals; and other items, the standards of which shall
be defined by the Department of Health. The
What products does the Milk Code cover? lactation station shall not be located in the toilet.
 Breast milk substitutes, including infant formula and
milk supplements.
 Foods, beverages, and other milk products (when
marketed or represented to be suitable, with or
without modification, for use as partial or total
replacement for breast milk).
 Bottle-fed complementary foods.
 Feeding bottles and teats.

POLICIES
1. Exclusive breastfeeding is for infants from 0 to 6
months
2. Breast milk has no substitute or replacement
NOTE: Breastfeeding is best for babies ESPECIALLY
during disasters
3. In addition to breastfeeding, appropriate and safe
complementary feeding of infants should start from
6 months onwards.
4. Breastfeeding is still appropriate for children up to 2
What Can Health Workers Do?
years of age and beyond.
 Remove poster’s that advertise formula or baby
5. Infant or milk formula may be harmful to a child’s
cereal.
heath and may damage a child’s health and may
 Refuse to accept free gifts or supplies of formula.
damage a child’s formative development.
 Refuse to allow free samples, gifts, leaflets to be
6. Other related products such as teats, feeding
given to mothers.
bottles, and artificial feeding paraphernalia are
 Eliminate teaching of formula use to every mother.
prohibited in health facilities.
Breastfeeding Counselling
RULES ON DONATIONS
 Breastfeeding gives children the best start in life.
 Donation of products and materials defined and
 It is estimated that over one million children die
covered by the Milk Code shall be strictly prohibited.
each year from diarrhea, respiratory and other
 Other donations which are given in kind or in cash
infections because they are not adequately
by milk companies, their agents, and their
breastfed.
representatives, must be coursed through the Inter-
 Many more children suffer from unnecessary illness
Agency Committee (IAC) for approval.
that they would not have if they were breastfed.
 V. Prohibitions/ Violations
 Breastfeeding also helps to protect mothers’ health. Carbohydrates
 The main carbohydrate is the special milk sugar
Global and National Recommendations for Infant lactose, a disaccharide. Breastmilk contains about
and Young Child Feeding 7 g lactose per 100 ml, which is more than in most
 Exclusive breastfeeding for 6 months; other milks, and is another important source of
 Introduce nutritionally adequate and safe energy.
complementary foods after the infant reaches 6 Protein
months of age;  Breast milk protein differs in both quantity and
 Continuing to breastfeed for 2 years or beyond quality from animal milks, and it contains a balance
of amino acids which makes it much more suitable
What is Exclusive Breastfeeding? for baby.
 Give an infant only breast milk, with the exception  The concentration of protein in breast milk (0.9 g
of drops or syrups consisting of vitamins, mineral per 100 ml) is lower than in animal milks.
supplements, or drugs. Vitamins and minerals
 No food or drink other than milk-not even water.  Breast milk normally contains sufficient vitamins for
an infant, unless the mother herself is deficient
 The exception is vitamin D. the infant needs
exposure to sunlight to generate endogenous
vitamin D – or, if this is not possible, a supplement.
 The minerals iron and zinc are present in relatively
low concentration, but their bioavailability and
absorption is high.
Anti-infective factors
 Breast milk contains many factors that help to
protect an infant against infection.
 Immunoglobulin, principally secretory
immunoglobulin.
 A (slgA), which coats the intestinal mucosa and
prevents bacteria from entering the cells;
 K white blood cells which can kill micro-organisms;
 K whey proteins (lysozyme and lactoferrin) which
can kil bacteria, viruses and fungi;
 K oligosaccharides which prevent bacteria from
attaching to mucosal surfaces.
Other bioactive factors
 Bile-salt stimulated lipase facilities the complete
Variations in the Composition of Breast Milk digestion of fat once the milk has reached the small
Colostrum intestine.
 is the breast milk that women produce in the first  Epidermal growth factor stimulates maturation of
few days after delivery. It is thick and yellowish or the lining of the infant’s intestine, so that it is better
clean in color. able to digest and absorb nutrients, and is less
Mature milk easily infected or sensitized to foreign proteins.
 is the breast milk that is produced after a few days. Colostrum and mature milk
The quantity becomes larger, and the breasts feel  Colostrum is the special milk that is secreted in the
full, hard and heavy. first 2-3 days after delivery.
Foremilk  It is produced in small amounts, about 40-50 ml on
 is the milk that is produced early in feed the first day.
Hindmilk  Colostrum in rich in white cells and antibodies.
 is the milk that is produced later in a feed. It looks  Colostrum provides important immune protection to
whiter than foremilk, because it contains more fat. an infant when he or she is first exposed to the
This fat provides much of the energy of a micro-organisms in the environment, and epidermal
breastfeed. growth factor helps to prepare the lining of the gut
Foremilk to receive the nutrients in milk.
 looks bluer than hind milk. It is produced in larger  Milk starts to be produced in larger amounts
amounts, and it provides plenty of protein, lactose, between 2 and 4 days after delivery, making the
and other nutrients. breasts feel full; the milk is then said to have “come
in”.
Physiological Basis of Breastfeeding  On the third day, an infant is normally taking about
Breast-milk composition 300-400 ml per hours, and on the fifth day 500-800
 Breast milk contains all the nutrients that an infant ml.
needs in the first 6 months of life, including fat,  From day 7 to 14, the milk is called transitional after
carbohydrates, proteins, vitamins, minerals and 2 weeks it is called matured milk.
water
Fats
 Breast milk contains about 3.5 g of fat per 100 ml of
milk, which provides about one half of the energy
content of the milk.
Results of Poor Attachment:

Hormonal Control of Milk Production

Terms for Infant Feeding


Exclusive breastfeeding
 means giving a baby no other food or drink,
including no water, in addition to breastfeeding
(except medicines and vitamin or mineral drops;
expressed breastmilk is also permitted).
Predominant breastfeeding
 breastfeeding a baby, but also giving a small
amounts of water or water-based drinks – such as
tea.
Full breastfeeding
 breastfeeding either exclusive or predominantly
Bottle feeding
Signs and Sensations of an Active Oxytocin Reflex
 feeding a baby from a bottle, including expressed
breastmilk.
Artificial feeding
 giving a baby some breastfeeds, and some artificial
feeds, either milk or cereal, or other food.
Timely complementary feeding
 giving a baby other food in addition to breastfeeding,
when it is appropriate, after the age 4-6 months.
How does the mother hold her baby?

How does the mother hold her breast?


Syringe Breast Pump

---------------------------------------------------------------------------------

PERINEAL FLUSHING
 is a hygienic care that involves cleaning the
perineum and genitalia.

Perineum
 is the area between the thighs and from the anterior
pelvis to the anus.
 Female-it is the area between vagina and
rectum.
 Male-it is between the scrotum and anus.

Purposes:
1. To remove normal perineal secretions and odors
2. To promote client comfort.
3. To prevent infection or contamination from the
rectum.

Indications:
 Patient in labor and post partum.
 Gynecological patient’s with perineal repair.
 Comatose
 Diabetic Patient
 Patient with indwelling catheter

Equipment:
1. Solution bottle
2. Pitcher
3. Clean gloves
4. 2 kidney basins
5. 7 pcs. dry cotton balls
6. 7 pcs. cotton balls soaked in cleansing solution
7. Bedpan
8. News paper
9. Rubber sheet
10. Draw sheet
11. Forcept
a) Pick-up 18. Using 7 dry cotton balls, dries the perineum with the
b) Working same stroke as above.(procedure #7-#14)
12. Perineal pad R: Moisture support the growth of many microorganism.
a) Maternity napkin 19. Remove the bedpan gently and turn the patient
b) Adult diaper to side. If necessary, takes one cotton ball and cleans
13. Bath blanket one side of the buttocks. Then clean the other side of
buttocks and dry both buttocks.
Procedure: R: To provide comfort.
1.Prepare all the needed materials. 20. Turn patient on her back and make her comfortable.
R: To save time and effort. 21. Sprays with antiseptic as needed.
2.Washes hands. R: To promote healing.
R: As universal pre-caution. 22. Puts the perineal pad with an up down motion, as
3. Prior to performing the procedure, introduce self and needed.
verify the client’s identity using agency protocol. Explain R: To prevent contamination of vagina and urethra from
to the client what you are going to do, why it is anal area.
necessary. 23. Does after care of the materials used. Remove and
R: To gain client’s trust and cooperation, as well as to discard gloves. Perform hand hygiene.
build rapport. 24. Record the treatment done to patient and any
4. Screen the patient or lock the door of unusual findings such as redness , excoriation, skin
the patient’s room. breakdown, discharge or drainage, and any localized
R: For the client’s comfort and privacy. areas of tenderness.
5. Wear gloves R: For documentation purposes that procedure was
R: To avoid cross contamination. done and means of communication.
6. Position the patient on dorsal recumbent (back-lying
position with the knees flexed and spread well apart) --------------------------------------------------------------------
R: Allows visualization on the perennial area.
(Cover her body and legs with the bath blanket
positioned so a corner is at her head, the opposite ENEMA (Labatiba)
corner at her feet , and the other two on the sides.)  is the instillation of a solution into the rectum and
7. Place the rubber sheet/draw sheet and bed pan under sigmoid colon.
the patient's buttocks .
R: Prevents the bed from becoming soiled. Purposes:
8. Drape the patient, exposing only the part to be 1. The primary reason for an enema is to promote
cleansed. defecation by stimulating peristalsis.
R: Minimum exposure lessen the embarrassment and 2. Vehicle for medications that exert a local effect on
provide warmth. rectal mucosa.
9. Inspect the perineum then flush the perineal area with 3. Used most commonly for the immediate relief of
warm water. constipation.
R: Note particular areas of inflammation, swelling 4. Emptying the bowel before diagnostic tests or
excessive discharge or secretions from the orifice and surgery, and beginning a program of bowel training.
presence of odor. Warm water is used to prevent chilling
and soften or dissolve blood clots. Types of Edema:
10. Get 7 pcs of cotton balls soaked in a cleansing 1. Cleansing Enemas
solution using pick up forceps.  promote the complete evacuation of feces from
R: To maintain sterility. the colon. They act by stimulating peristalsis
11. Cleanses external genitalia starting from midline of through the infusion of a large volume of
symphysis pubis down to the anus. Never retrace a solution or through local irritation of the
stroke. mucosa of the colon.
R: To prevent contamination of the area.  Common solution for cleansing enemas
12. With the second cotton ball cleans starting from  Tap water
mons pubis in figure of 7 by way of external labium  after infusion into the colon, tap water
towards the anus. Discard. escapes from the bowel lumen into
R: Secretions that tend to collect around the labia interstitial spaces. Use caution if
minora facilitate bacterial growth. ordered to repeat tap-water enemas
13. Do likewise on the opposite side with the because water toxicity or circulatory
next cotton ball. overload develops if the body
14. With the fourth cotton ball cleans starting absorbs large amounts of water.
from mons pubis in figure of 7 by way of internal  Normal saline solution
labium towards the anus. Discard.  the safest solution to use because it
15. Do likewise on the opposite side with the next exerts the same osmotic pressure as
cotton ball. fluids in interstitial spaces
16. With another cotton ball, cleans groin- starting from surrounding the bowel.
groin going up the thigh . Use another side of the same  Soapsuds solution
cotton ball for near groin.  add soapsuds to tap water or saline to
17. Flush perineum thoroughly with sterile warm water. create the effect of intestinal irritation
to stimulate peristalsis.
 Hypertonic solution
 the colon fills with fluid, and the 7. Basin, washcloths, towel, and soap
resultant distention promotes 8. IV pole
defecation for patients who are 9. Enema container with tubing and clamp attachment
dehydrated and young infant. 10. Appropriate-size rectal tube:
Commercially prepared Fleet enema a) Size of Rectal Tube
is the most common. i. Adult - Fr.# 22-30
2. Oil Retention ii. Children - Fr.# 14-18
 lubricate the feces in the rectum and colon. The iii. Infant - Fr.# 12
feces absorb the oil and become softer and b) Correct Volume of the Warmed Solution:
easier to pass. To enhance action of the oil, Adult Children Infant
the patient retains the enema for several hours Amount of 500-1,000 250 - 500 250ml or
if possible. Solution ml ml less
3. Carminative enemas Distance of 7.5 - 10 cm 5 - 7.5 cm 2.5 - 3.75
 provide relief from gaseous distention. They Tube Insertion (3-4 in) (2-3 in) cm (1-1.5
improve the ability to pass flatus. in)
4. Medicated enemas contain drugs Solution 40.5 - 43 C 37.7 C
 sodium polystyrene sulfonate (Kayexalate), Temperature
used to treat patients with dangerously high
serum potassium levels. “Enemas until clear”
 medicated enema is neomycin solution, an  order means that you repeat enemas until patient
antibiotic used to reduce bacteria in the colon passes fluid that is clear of fecal matter.
before bowel surgery
 enema containing steroid medication may be Assessment:
used for acute inflammation in the lower colon. 1. Assesses status of patient: last bowel movement,
normal versus most recent bowel pattern, presence
Precautions: of hemorrhoids, mobility, and presence of
 Enemas should not be used as a first line treatment abdominal pain.
for constipation. R: Determines factors indicating need for enema and
 Never deliver more than three consecutive enemas influencing type of enema used. Also establishes a
to treat a patient. Frequent use of enemas can lead baseline for bowel function.
to fluid overload, bowel irritation, and loss of muscle 2. Review medical record for presence of increased
tone of the bowel and anal sphincter. intracranial pressure, glaucoma, or recent
 A patient with diarrhea may not be able to hold an abdominal, rectal, or prostate surgery.
enema. R: These conditions contraindicate use of enemas.
 Must be used with caution in cardiac patients who 3. Inspect abdomen for presence of distention.
have arrhythmias or have had a recent myocardial R: Provides a baseline for determining effectiveness of
infarction. the enema.
 Insertion of the enema tube and solution can 4. Determine patient’s level of understanding of
stimulate the vagus nerve which may trigger an purpose of enema.
arrythmias such as bradycardia. R: Allows you to plan for appropriate teaching measures.
 Enemas should not be given to patients with 5. Review the health care provider’s order for type of
undiagnosed abdominal pain because the enema and number to administer.
peristalsis of the bowel can cause an inflamed R: Enemas requires a health care provider’s order.
appendix to rupture. Determines number and type of enema you will give.
 Should be used cautiously in patients who have
had recent surgery on the rectum, bowel, or Planning:
prostate gland. 1. Collect appropriate equipment.
 If the patient has rectal bleeding or prolapse of 2. Identify patient using two identifiers (name and
rectal tissue from the rectal opening, cancel the birthday or name and medical record number)
enema and consult with the physician before according to agency policy.
proceeding. R: Ensures correct patient. Complies with the Joint
 Do not force the enema catheter into the rectum Commission standards and improves patient safety.
against resistance. This can cause trauma to the 3. Assemble enema bag with appropriate solution and
rectal tissue. rectal tube if enema administration set does not
 Use only mild castile soap (hard white unperfumed have tube integrated kit.
soap made from olive oil and lye) for soapsuds R: The proper equipment promotes the best outcome
enemas because other soap preparations are too from the procedure.
harsh and irritate the rectal tissue.
Implementation:
Equipment: 1. Perform hand hygiene and apply clean gloves.
1. Clean gloves R: Reduces transmission of microorganisms.
2. Water-soluble lubricant 2. Provide privacy by closing curtains around bed or
3. Waterproof, absorbent pads closing door.
4. Bath blanket R: Reduces embarrassment for patient.
5. Toilet tissue
6. Bedpan, bedside commode, or access to toilet
3. Raise bed to appropriate working height for nurse: 9. Hold tubing in rectum constantly until end of fluid
stand on right side of bed and raise side rail on instillation.
opposite side. R: Bowel contraction causes expulsion of rectal tube.
R: Promotes good body mechanics and patient safety. 10. Open regulating clamp and allow solution to enter
4. Help patient into left side-lying (Sims’) position with slowly to while holding container at patient’s hip
right knee flexed. Children may also be placed in level.
dorsal recumbent position. R: Rapid instillation stimulates evacuation of rectal tube.
R: Positioning allows enema solution to flow downward 11. Raise height of enema container slowly to
by gravity along natural curve of sigmoid colon and appropriate level above anus: 30 - 45 cm (12-18 in)
rectum, thus improving retention of solution. for height enema, 30 cm (12in) for regular enema
5. Place waterproof pad under hips and buttocks. 7.5 cm (3 in) for low enema. Instillation time varies,
R: Prevents soiling of linen. depending on volume of solution you administer
6. Cover patient with bath blanket, exposing only (e.g., 1 L/10min).
rectal area, clearly visualizing anus. Separate R: Allows for continuous, slow instillation of solution;
buttocks and examine perianal region for raising container too high causes rapid instillation and
abnormalities, including hemorrhoids, anal fissure, possible painful distention of colon.
and rectal prolapse (protrusion of the colon through 12. Lower container or clamp tubing if patient
the anal opening). complaints of cramping or if fluid escapes around
R: Provides warmth, reduces exposure of body parts, rectal tube.
and allows patient to feel more relaxed and comfortable. R: Temporarily stopping instillation prevents cramping,
Findings will influence approach to insert enema tip. An which prevents patient from retaining all fluid, altering
enema is contraindicated if there is a prolapse. the effectiveness of enema.
7. Place bedpan or commode in easily accessible 13. Clamp tubing after you instill all solution.
position. If patient will be expelling contents in toilet, R: Prevents air from entering rectum.
ensure that toilet is free. (if patient will be getting up 14. Place layers of toilet tissue around tube at anus and
to go to bathroom to expel enema, place his or her gently withdraw rectal tube.
slippers and bathrobe in easily accessible position) R: Provides for patient’s comfort and cleanliness.
R: Try to avoid incontinence of the stool and enema fluid 15. Explain to patient that a feeling of distention and
to avoid discomfort and psychological stress. some abdominal cramping are normal. Ask patient
8. Administer enema to retain solution as long as possible while lying
a) Enema bag quietly in bed. (for infant or young child, gently hold
i. Add warmed solution to enema bag: warm buttocks together for few minutes.)
tap water as it flows from faucet, place R: Solution distends bowel. Length of retention varies
saline container in basin of hot water with type of enema and patient’s ability to contract rectal
before adding saline to enema bag and sphincter. Longer retention promotes more effective
check temperature of solution by pouring stimulation of peristalsis and defecation.
small amount of solution over inner wrist. 16. Discard enema container and tubing in proper
If soapsuds enema is ordered, add castile receptacle or rinse bag out thoroughly with warm
soap. soap and water if container is reusable.
R: Hot water will burn intestinal mucosa. Cold water R: Reduces transmission and growth of microorganism.
causes abdominal cramping and is dificult to retain. 17. Help patient to bathroom or help to position patient
ii. Raise container, release clamp, and allow on bedpan.
solution to flow long enough to fill tubing. R: Normal sitting position promotes defecation.
R: Removes air from tubing. 18. Help patient as needed to wash anal area with
iii. Reclamp tubing. warm water, premoistened perineal wipe, or no-
R: Prevents further loss of solution. rinse perineal cleanser. (if you administer perineal
iv. Lubricate 6 to 8 cm (2.5 to 3 inches) of tip care, use clean gloves.)
of rectal tube with water-soluble R: Fecal contents irritate skin. Hygiene promotes
lubricating jelly. patient’s comfort.
R: Allows smooth insertion ot rectal tube without risk for 19. Remove and discard gloves and perform hand
irritation or trauma to mucosa. hygiene.
v. Gently separate buttocks and locate anus. R: Reduces transmission of microorganisms.
Instruct patient to relax by breathing out
slowly through mouth. Evaluation:
R: Breathing out promotes relaxation of external anal 1. Observe character of feces and solution evacuated.
sphincter. (Caution patient against flushing toilet before inspection.)
vi. Insert tipof enema tube slowly by pointing Inspect color, consistency, amount of stool, odor, and
tip in direction of patient's umblicus. fluid passed.)
Length of insertion varies: adult and 1% R: Determines if stool is evacuated or fluid is retained.
adolescent: 75 to 10 cm (3 to 4 inches); Note abnormalities such as presence of blood or mucus.
child: 5 to 7.5 cm (2 to 3 inches); infant: 2. Assess condition of abdomen, cramping, rigidity, or
25 to 3.75 cm (1 to 1.5 inch). distention indicates a serious problem.
R: Careful insertion prevents trauma to rectal mucosa R: Determine if distention is relieved. Excess volume
from accidental lodging of tube against rectal wall. distends or damages the bowel.
Insertion beyond proper limit causes bowel damage.
Unexpected Outcomes and Related Intervention
 Abdomen becomes rigid and distended, and patient g) assess whether a sterile specimen needs to be
complains of severe pain. collected.
 Stop enema 3. Determine the most appropriate method of
 Notify Doctor catheterization based on the purpose and any criteria
 Obtain vital signs specified in the order to be used.
 Abdominal pain or cramping develops. 4. Use a straight catheter if only a spot urine is needed,
 Slow rate of instillation, have patient take slow if amount of residual urine is being measured, or
deep breaths temporary decompression/emptying of the bladder is
 Bleeding develops required.
 Stop Enema 5. Use an indwelling/retention catheter if the bladder
 Notify Doctor must remain empty or continuous urine
 Remain with patient and obtain vital signs measurement/collection is needed.
6. Availability of equipment.
Documentation:
1. Record type and volume of enema given Types of Catheter
2. Time administered Short-term (intermittent) catheters
3. Characteristics of results  A condom catheter, which fits on the outside of the
4. Patient’s tolerance to the procedure penis using adhesive, can be used for short-term
5. Report failure of patient to defecate and any catheterization in males.
adverse effects.  SUPRAPUBIC CATHETERS
 basically an indwelling catheter that is
-------------------------------------------------------------------- placed directly into the bladder through
the abdomen. The catheter is inserted
above the pubic bone. The insertion site
Urinary Catheterization (opening on the abdomen) and the tube
 is the introduction of a catheter through the urethral must be cleansed daily with soap and
meatus into a patient's bladder. water and covered with a dry gauze.
Long-term use
Purposes:  A catheter that is left in place for a period of time
Therapeutic may be attached to a drainage bag to collect the
1. To relieve acute or chronic urinary retention. urine.
2. Instillation of medication. Indwelling catheterization
3. To drain urine pre-operatively and post-operatively.  Providing relief for persons with an initial episode of
4. To assess the amount of residual urine if the bladder acute urinary retention, allowing their bladder to
empties incompletely. regain its normal muscle tone.
5. To provide for intermittent or continuous bladder  Provide palliative care for incontinent persons who
drainage and irrigation. are terminally ill or severely impaired.
6. To prevent urine from contacting an incision after
perineal surgery. Sizing scale of the French catheter system:
Diagnostic Infant : Fr 5-6
1. To aseptically secure an uncontaminated sample of Children : Fr 8-10
urine for microbiological examination. Adult Female : Fr 14-16
2. To facilitate accurate measurement of urinary output Adult Male : Fr 16-18
for critically ill patients. Note:
 Larger catheters greater than Fr 16 can distend the
Contraindications: urethra, permanently damage the urethra and
1. Actual or suspected urethral trauma (e.g. pelvic bladder neck, and cause bladder spasms and
trauma). leaking around the catheter.
2. Urethral stricture or obstruction  Smallest size catheter possible to minimize trauma
3. Urethral trauma and promote adequate drainage of the periurethral
4. Bladder trauma glands. This decreases the risk of infection.
5. Urethral / bladder spasm
6. Pressure necrosis of the urethra Planning:
7. UTI 1. Prepare needed equipment
a) Sterile Pack
Assessment: i. A pack which contains:
1. Verify doctor’s order. ii. Kidney basin
2. Assess the status of the patient. iii. 3-5 sterile cotton balls
a) when the patient last voided. iv. sterile drape or fenestrated sterile drape
b) level of awareness or the developmental stage v. sterile forcep
of the patient. vi. catheter set
c) mobility and physical limitations of the patient. vii. urometer bag
d) client’s age viii. sterile specimen bottle/container
e) bladder distention ix. Prefilled 10 cc sterile syringe
f) presence of any pathological conditions that x. 5-10 cc syringe
may impair the passage of urine. xi. KY Jelly
b) Non sterile 9. Open the set. Observing sterile technique.
i. Rubber sheet/waterproof pad/bed 10. Put on the sterile gloves observing the proper
protector technique .
ii. Drop light/flashlight R - To prevent contamination and to maintain the sterility
iii. Waste receptacle of the set.
iv. Bath blanket 11. Put the eye sheet.
v. Adhesive/hypoallergenic tape 12. Lubricate the catheter for about 1 1/2 inches being
careful not to plug the eye of the catheter.
Equipment: R - Lubrication reduces friction and prevents urethral
1. Disposable sterile catheter trauma thus reducing pain upon insertion.
2. Antiseptic solution (sterile cotton balls with 13. Pick-up the catheter at least 3 inches from the tip of
betadine) the thumb. Place the thumb and one finger of the left or
3. OS right hand between the labia minora, separate and pull
4. Pick-up forceps or dressing forceps up.
5. Sterile lubricant (single use sachet) R - Separating the area surrounding the meatus
6. 10 cc syringe with sterile water (for retained provides visibility.
catheter)
7. Hypoallergenic plaster
8. 2 Sterile Gloves
9. Urometer Bag

Different Catheters:

14. Insert the catheter 2-3 inches or until urine


begins to flow.
R - Female urethra is approximately 1 ½ - 2 ½ inches.
15. Encourage patient to relax (slow deep breath) and
insert as the client exhales for 2-3 inches or until urine
flows.
Implementation: R- Relaxation of the external sphincter aids in easy
Female Urinary Catheterization: insertion of the catheter.
1. Check the doctor's order for catheterization. 15. If a specimen is to be collected, pinch the catheter
R - To ensure that you have the right patient and right with the thumb and the first finger while placing the
procedure to be done. specimen container in position.
2. Wash before preparing the set R - Urine will flow out of the catheter if lumen is not
R - Reduces microorganisms that causes disease and occluded.
contamination. 16. Rest the hand in the pelvic area to hold the catheter
3. Identify the patient and explain the procedure patient. in place to prevent pulling and pushing the Catheter.
R - The patient will feel reassured if the procedure is R - Withdrawing the catheter and pushing it back in the
explained and if she is handled gently and considerately. urethra increases the possibility of contamination to the
4. Provide privacy to the patient by screening or closing urethra.
the room and position patient position in a dorsal 17. Connect the catheter to the urine bag and be sure
recumbent position with the knees flexed. the bag is close and kept the connection sterile.
R - Good visualization of the meatus is essential to R- So that the urine will not leak from the bag and
introduce the catheter. Gravity will aid the flow of the maintaining its sterility.
urine when bladder is higher than the end of the catheter. 18. Inflate the balloon 10cc of sterile water using 10cc
5. Drape the patient exposing only the area to be used. syringe.
R - Embarrassment and chilliness will make the client to R - Making it sure that there will be no leakage of water
become tensed and tension may interfere with the easy from the balloon thus it serves the purpose of anchoring
introduction of the catheter. into the bladder.
6. Apply direct light for visualization of the genital area. 19. Secure the indwelling catheter by taping it to the
R - Placing the equipment in order will increase speed of patient's thigh. Allow some slack of the tubing to
performance. Reaching over sterile items increases risk accommodate the patient's movement.
of contamination. R - To secure the catheter properly. Slack prevents
7. Open the tray and arrange equipment for tension on the catheter when the patient moves.
convenience and accordingly to avoid contamination of 20. Place the patient comfortably.
the sterile items (always observe aseptic technique). R- This fosters relaxation and comfort.
R - Placing the equipment in order will increase speed of 21. Do the aftercare. And record the time f procedure
performance. Reaching over sterile items increases risk done, amount and color initial output.
of contamination. R- Cleaning the environment promotes comfort to the
8. Using the sterile forceps, cleanse the area using client. Moisture is a good environment for bacterial
cotton ball with betadine. growth.
22. Wash hands  Initial uterine contractions are short and mild,
R- Handwashing prevents the spread of microorganisms. but frequency, intensity, duration increase.
During transition stage, the cervix completes
Labor and Delivery dilation and stretches over baby's head. The
entire process usually lasts from 4- 18 hours
(Early Essential Newborn Care) 2. Mucous plug is dispelled
 The mucous plug falls out as cervix dilates; it is
Statistics accompanied by a tinge of blood, and is
 3.4 million pregnancies occur every year referred to as “bloody show”.
 11 mothers die of pregnancy - related causes 3. Amniotic sac breaks
everyday  Before labor begins, or in the early stages, the
 Leading cause of maternal deaths: amniotic sac ruptures under pressure and fluid
 PPH trickles or gushes out. Sometimes the sac
 HPN d/o must be broken manually.
 Abortion related complications
 40, 000 newborns die each year from causes that
are most preventable such as complications of:
 Prematurity (41%)
 Birth asphyxia (15%)
 Severe infection (16%)

National Policies for Maternal and Newborn Care


 The Philippine government, along with the
international community, has made legislative
efforts to ensure that care of a certain standard is
accessible to all mothers and babies. Stages of Labor

Health Policies for Essential Intrapatom and


Newborn Care (EINC)
 The Department of Health has issued directives to
ensure the highest quality of care for mothers and
their newborns.

DOH Policies
 Revitalization of the Mother-Baby Friendly Hospital
Initiative in Health Facilities with Maternity and
Newborn Care Services (Administrative Order No.
2007-0026).
 Main objective is to transform all health
institutions with maternity and newborn
services into facilities that fully PROTECT,
PROMOTE AND SUPPORT rooming – in,
breastfeeding and mother-baby friendly Simulation
practices. 1. A 27 year old, gravida 3, para 2, at 38 weeks
 Implementing Health Reforms for the Rapid gestation presents to labor and delivery
Reduction of Maternal and Neonatal Mortality complaining that she thinks she is in labor.
(Administrative Order No. 2008-0029). WHAT ARE KEY ELEMENTS IN THE HISTORY?
 Adopting New Policies and Protocol on Essential • Last menstrual period?
Newborn Care (Administrative Order No. 2009- • Estimated date of delivery?
0025). • Loss of fluid?
 Issued on December 7, 2009 • Fetal movement?
 Outlines specific policies & principles to follow • History of infections?
for all health care providers involved in • Past medical history
newborn health care. • Social history
 The Aquino Health Agenda: Achieving Universal • Pregnancy and obstetric history
Health Care for All Filipinos (Administrative Order • Vaginal bleeding?
No. 2010-0036). • Contractions?
• Group B streptococcus status?
4 Steps to Save Newborn Lives: • Past surgical history
 Immediate and thorough drying of the newborn. WHAT ARE KEY ELEMENTS IN THE PHYSICAL?
 Early skin-to-skin contact b/n mother & newborn. • Vital signs
 Properly timed cord clamping and cutting. • Abdominal examination
 Non-separation of newborn and mother for early • Speculum examination
breastfeeding. • Cervical examination
• Leopold
3 Signals Indicate that Labor has Begun:
1. Contractions begin History:
 Last menstrual period 39 weeks ago  Negatives
 Uncomplicated pregnancy to date, two prior full
term vaginal deliveries. Clue to Contractions
 Possible leaking, no bleeding, painful contractions D - Duration (How long)
every 8-10 minutes. I - Intensity (How strong)
 No history sexually transmitted infections. F - Frequency (How often)
 No significant past medical or surgical history, no
allergies. Uterine Contractions
Physical:  Palpation
 Vitals: blood pressure 90/60, heart rate 102 bpm,  Timing
respiratory rate 12, temperature 37.6 Centigrade  Frequency
 Abdomen: term uterus, fundal height 39 cm,  Duration
cephalic, otherwise soft, non-tender.  Strength
 Speculum examination: negative for pooling,
ferning, and nitrazine
 Hemoglobin 12.1 g/dL
 Blood type O+, antibody screen negative

Prenatal Care
 Visit frequency
 Every 4 weeks through 28 weeks
 Every 2 weeks through 36 weeks
 Every week thereafter
 Accomplish the following:
 Assess well being During Labor
 Ongoing education  No routine enema and shaving.
 Perform routine screening  Assist intravenous fluid therapy insertion.
 Routine measurements  Instruct patient and maintain on NPO only when
 Blood pressure, weight, uterine size, fetal heart indicated.
rate, urine dipstick
 • Routine lab Immediate Newborn Care (The First 90 minutes)
 HIV, hepatitis B, syphilis, chlamydia, gonorrhea, Time Band: At perineal bulging, with presenting part
urine culture visible (2nd stage of labor)
 Blood type, Rh status, antibody screen Preparing for Delivery:
 Complete blood count  Ensure that delivery area is draft-free and between
 Glucose screening at 24-28 weeks 25-28C using a room thermometer.
 Group B streptococcal screening at 35-37  Asks the woman if she is comfortable in a semi-
weeks upright position.
 Immunizations  Ensure the woman’s privacy.
 Influenza, pertussis  Wash hands with clean water and soap.
 Other considerations Preparing for Delivery Table:
 Genetic screening options  Prepare a clean resuscitation area on a firm flat
 Alpha fetal protein surface.
 Hemoglobin electrophoresis  Check if the equipment within easy reach.
 Cystic fibrosis screening  Check resuscitation equipment.
 Arrange all instruments on a clean delivery table in
Upon Arrival at Facility a linear sequence.
 Identify mothers in PRETERM LABOR at point of Perineal Bulging
entry.  Uncontrollable urge to push.
 Assess history  Clean the perineum with antiseptic solution
 Conduct physical exam  Perform proper HANDWASHING
 Check initial vital signs  Wear sterile gloves appropriately
 Obtain birth plan  Piu on sterile leggings
 Determine companion of choice  No routine episiotomy
 No fundal pressure
Early Essential Newborn Care: EENC
DURING LABOR: At the Time of Delivery and EINC
 Allow position of choice. Time Band: Within the 1st 30 seconds
 Allow mother to have oral fluids and light snacks. At the Time of Delivery:
 Monitor progress of labor utilizing Partograph.  Drape the mother’s abdomen with sterile towel in
preparation for drying the baby.
External Fetal Monitoring  Encourage the mother to push as desired.
 Two belts  Apply perineal support and do controlled delivery of
 Uterine contractions the head.
 Fetal heart monitoring  Call out time of birth and sex of the baby.
 Benefits  Inform the mother of the outcome.
 Ex.
 Baby out  If a baby is crying and breathing normally, avoid
 10:15 am any manipulation, such as routine suctioning, that
 Live baby boy may cause trauma or introduce infection.
 Place the newborn prone on the mother’s abdomen
or chest skin-to-skin.
 Cover newborn’s back with a blanket and head with
bonnet.
 Place identification band on ankle.

Time Band: If after 30 seconds of thorough drying,


newboen is not breathing or is gasping
 Reposition, suction and ventilate.
 Clamp and cut the cord immediately.
 Call for help.
Delivery of the Baby  Transfer to warm, firm surface.
 The mother may stand with her legs apart, squat,  Inform the mother that the newborn has difficulty
lean over, recline backward or lie down with legs breathing and that you will help the baby to breath.
supported to facilitate the delivery.  Start resuscitation protocol.
 https://www.youtube.com/watch?v=dYu-0rOnLpA
At the Time of Delivery and EINC  If the baby is non-vigorous (limp/floppy and not
Time Band: First 30 seconds breathing) and meconium-stained
 Dry and provide warmth.  Clear the mouth
 thoroughly dry the baby for at least 30 seconds  Start bag/mask ventilation
by wiping the eyes, face, head, front and back,  Refer and transport
arms and legs.
 While performing a quick check for color and Time Band: 1-3 minutes
breathing.  Give Oxytocin IM within one minute after baby’s
birth : BY ASSIST.
 Do delayed or non-immediate cord clamping.
 Remove the first set of gloves immediately prior to
cord clamping.
 Palpate the umbilical cord to check for pulsation,
have stopped (typically at 1-3 minutes).
 Clamp the cord with cord clip 2cm from the
newborn’s abdomen.
 Clamp with straight/curve clamp around the cord at
5 cm from the newborn’s abdomen.
At the Time of Delivery and EINC  Cut the cord between clamps with sterile scissors.
Time Band: 1-3 minutes  Observe for oozing blood.
 Note: Do not ventilate unless the baby is floppy/limp  Assess the completeness of arteries and vein.
and not breathing.
 Do not suction unless the mouth/nose are blocked Post-Delivery of the Baby
with secretions or other material. Time Band: 1-3 minutes
 Remove the wet cloth.  Note:
 Place the newborn skin-to-skin on the mother’s  Do not milk the cord towards the newborn.
abdomen or chest
 Cover the baby with a dry cloth and the baby’s Placental Stage
head with a bonet. 3rd Stage of Labor
 Dispose wet cloth properly. Placental Delivery
 Exclude a second baby by palpating the abdomen.  Contraction
 Do not separate the newborn from mother, as long  Globular uterus
as the newborn does not exhibit severe chest in-  Cord lengthens
drawing, gasping or apnea and the mother does not  Rush of vaginal blood
need urgent medical stabilization e.g. emergent
hysterectomy. Placental Stage
 Do not put the newborn on a cold or wet surface.  Wait for the strong uterine contractions
 Do not wipe off vernix if present.  Apply controlled cord traction on the uterus,
 Do not bathe the newborn earlier than 6 hours of continuing until the placenta is delivered.
life.  Massage the uterus until firm.
 Do not do foot printing.  Examine the placenta for completeness and
 If the newborn must be separated from his/her abnormalities.
mother, put him/her on a warm surface, in a safe  Dispose the placenta in a leak-roof container of
place close to the mother. plastic bag.

Time Band: If after 30 seconds of thorough drying, 4th Stage of Labor


newboen is breathing or crying
 Do skin-to-skin contact.
 Inspect the lower vagina and perineum for Intake and Output Measurement
lacerations and assist in repair as necessary.
 Measure for 24 hour period to assess for an actual
 Clean the mother by flushing the perineum and
or potential fluid imbalance.
apply perineal pad, diaper, and change gown.
 ml/cc- unit used to measure intake and output
 Check that the mother is comfortable.
 Check the doctors order
 Check the uterus is well-contracted.
 Provide the I&O record sheet at the bedside
 Advice the mother to maintain skin-to-skin contact
 Instruct the pt. or watcher that accurate
with baby prone on mother’s chest in between the
measurement is important.
breasts with head turned to one side.
 Advise the mother to observe for feeding cues and
Purposes:
cite examples of feeding cues.
1. To monitor the patient’s fluid status over a 24-hour
 Support the mother by instructing her on proper
period.
breastfeeding, positioning and attachment.
2. To evaluate the effectiveness of the diuretic or
 https://www.youtube.com/watch?v=dYu-0rOnLpA
rehydration therapy.
Episiotomy
Equipment:
 I and O sheet
 Calibrated cup or glass
 Graduated container for output
 Bedpan or urinal
 Working gloves
 Pen

Clients at Risk for Fluid and Electrolyte Imbalance


1. Elderly, very young, handicapped.
2. Gained or lost more than 5 lbs per week.
3. NPO clients.
4. Clients receiving diuretics.
5. Excessive fluid losses and requiring increased
intake.
6. Patients with chronic diseases such as CHF, DM,
liver disease, Kidney and Lung diseases, CA
7. Post-operative clients.
8. Confused clients or those with altered level of
consciousness who may not be able to
Time Band: Within 90 minutes of age communicate needs or respond to thirst.
 Provide support for initiation of breastfeeding.
 Observe the newborn. Only when the newborn Intake Includes:
shows feeding cues (e.g. opening of mouth,  Oral fluids
tonguing, licking,  Ice chips
 rooting).  Foods that are or tend to become liquidat room
 Move her newborn onto her breast, aiming the temperature.
infant’s lower lip well below the nipple.  Tube feedings
 Do eye care.  Parenteral fluids
 Administer erythromycin or tetracycline ointment or  Intravenous medications
2.5% povidine-iodine drops to both eyes after  Catheter or tube irrigants
newborn has located breasts.  Transfused blood product
 Do not wash away the eye antimicrobial.
Output Includes:
Time Band: Within 90 minutes of age to 6 hours  Urine
 Give Vitamin K prophylaxis.  Vomitus and liquid feces
 Inject a single dose of Vitamin K 1 mg IM.  Diaphoresis
 Inject Hepatitis B vaccine intramuscularly and BCG  Tube drainage
intradermally.  Wound drainage
 Thorough examine the baby.  Rapid, deep respiratory rate
 Weigh the baby and record.
 Carefully get the anthropometric measurements of
the newborn starting with head circumference,
chest circumference, abdomen circumference and
the newborn’s length.
 Takes newborn’s vital signs.
 Check the birth injuries, malformations or defects.

--------------------------------------------------------------------------------
Sample Question:
1. Pt. Susan has an IVF of D5LR1L @ 700cclevel. At
10am she voided twice with250cc then she took a
glass of juice about 300cc. After an hour, she
vomits about 110cc. Then, she took a cup of soup
with 180cc. She voided again about 200cc. At the
end of your shift, her IVF consumed and
discontinued. Compute her intake and output.
Answer:
Intake:
Oral-480cc
IVF- 700cc
1,180cc

Output:
Urine- 450cc
Emesis-110cc
560cc

2. Pt. Samuel has NGT feeding q 4hrs in a shift with


300cc per feeding plus 30cc sterile water every
after feeding. He has an IVF of D5NM1L @ 800cc
received level. He has a urinary catheter attached
to urobag. At 1pm, the SN drained his urine bag
collector with an output of 550cc. Within your shift,
he defecated twice with the remaining IVF level of
400cc. What is the intake and output of the pt.
Answer:
Intake:
IVF- 400cc
NGT 660cc
1,060cc

Output:
Urine- 550cc
BM- 2x
550cc

You might also like